Compilation 509+ MCAT Study Habits

This forum made possible through the generous support of SDN members, donors, and sponsors. Thank you.

supremus

Full Member
7+ Year Member
Joined
Jan 13, 2015
Messages
86
Reaction score
77
Use the template below:

1) Your individual scores and composite score
2) The study method used for each section
3) What materials you used for each section(Kaplan, TPR, Examkrackers, AAMC, TBR, etc)
4) Which practice tests did you use? (Optional: include scores)
5) What was your undergraduate major?
6) Any other tips you may have for those of us who still have this test lurking over us?
7) How long did you study for the MCAT?

Members don't see this ad.
 
Last edited:
  • Like
Reactions: 28 users
Solution
Use the template below:

1) Scores:
  • C/P: 128
  • CARS: 131
  • Bio: 132
  • Psych: 127
  • Total: 518
2) The study method used for each section
  • Overall: I read every page of the review books and took notes for chem and psych, as they were my weakest sections. For any section that I had never gone over in school, I read and took notes (such as the metabolism sections). I would go back and forth with the Kaplan online videos/lessons, their practice problems, and the book in order to really memorize the info
  • Chem/Phys: this was easily my weakest section to begin with, so I went back to the basics (for chem)
  • CARS: the Kaplan tips on how to work with the CARS section were invaluable. Their hour-long MCAT channel...
1) Score: 524 (99th Percentile); 132 Chem/Phys, 131 CARS, 132 Bio, 129 Psych/Soc
2) Study Method: Content review by prep books for the first two months, (roughly one chapter per day) including all of the chapter questions. Last month: reading through notes and then taking each of the 30 min unit tests provided and reviewing chapter questions from before. Last week: turning all of the important topics into flashcards and reviewing ad nauseum.
3) Materials: ExamKrackers and the AAMC Flashcards
4) Practice tests: Just the official aamc practice exam (514 - took two weeks before MCAT) and the unscored aamca practice exam (~520 - one week before MCAT). I had already taken the MCAT a year and a half ago, so I was pretty sure how good my baseline was (33 - 11/11/11).
5) Undergrad major: Double major, Molecular and Cellular Biology and Mesopotamian History.
6) Tips:

TEACH IT TO SOMEONE ELSE. For any subject you don't understand that well, or can't quite fully grasp, explain/teach it to someone else. It is 100% the best way to really understand a topic and be able to apply it on the test.
More than knowing the topics, the test really makes you use critical thinking and understanding of whole concepts and how they relate to systems as a whole. Teaching it to someone else makes it so much easier to apply what you know on the test.

7) How long: Little under three months. I was also working full time.
 
  • Like
Reactions: 2 users
1) Your individual scores and composite score
Total: 512 PS: 127 CARS: 128 BIO:129 PSY: 128
2) The study method used for each section

This time I used Nymeria's MCAT Plan adapted from SN2ed which consists of one chapter of info and practice of each subject (Chem, Bio, Ochem, etc) a week and intense practice. I also took Florida Atlantic University's MCAT prep course but did not find it very useful.
3) What materials you used for each section(Kaplan, TPR, Examkrackers, AAMC, TBR, etc)
I used mostly Berkely Review for Chemistry, Physics and OChem and supplemented with Princeton Review. For Biology I used ExamKrackers. For CARs I used a combination of Princeton, NextStep, and Examkrackers. For psychology I used Princeton Review.
4) Which practice tests did you use? (Optional: include scores)
I took Princeton Review practice tests, Examkrackers CBT 1 and 4, Next Step Test 1,3, and 4, AAMC Sample test and AAMC Official Practice Test (I got a 506).
5) What was your undergraduate major?
Biological Science
6) Any other tips you may have for those of us who still have this test lurking over us?
I rushed to take the old MCAT without really studying before it changed and got 22/32nd percentile so I can not stress enough to practice! Use whatever material you can get your hands on, from EK 1001 to the full lengths available. Take your time to understand the information, make connections between subjects and see the overall picture.
7) How long did you study for the MCAT?
3 months

From a 22 (equal to a 497) to a 512!!! That is amazing. Congratulations on a great score and all of the work you put into doing it right.
 
  • Like
Reactions: 4 users
Members don't see this ad :)
1) Actual Score
516 (130/128/128/130)

2) Study Method

Four months before my exam, I began going through the Kaplan 7-Subject book set and reading each book cover-to-cover, taking notes as I went. I never actually used these notes, but summarizing what I was reading helped keep me engaged. I also did an occasional practice test. During this time, I live-streamed a few ThinkMCAT classes. I was also going to school full time, working, and volunteering.

A month before the exam, the semester was mostly over and I was able to just focus on studying. At this point, I had finished the content review and began focusing on the AAMC and Khan Academy materials and doing practice exams. I also made study sheets and had a friend quiz me on the material.

A week before the exam, I memorized everything in the Kaplan QuickSheet. I also watched more Khan Academy videos of concepts I was still just not understanding.

3) Materials (in order of usefulness)

Kaplan 7-Subject Review
This was my primary study tool. I used it to familiarize myself with the content.

Khan Academy Videos
Definitely important, but make sure you check the comments after watching a video because there are often mistakes.

AAMC Materials
I didn't find them as useful as many people say, but I used the practice questions and book until I felt that I had an idea of what to expect.

ThinkMCAT Classes (livestreamed)
For subjects you aren't comfortable with, this class will help you learn what is most important.

Princeton Review Materials
Used these books mostly for bio/biochem to cross-check what I should memorize, since Kaplan doesn't emphasize that.

4) Practice Tests (in no particular order)

Actual 6/2: 516 (130/128/128/130)

Next Step
FL 1 5/10: 510 (128/126/127/129)
FL 2 5/18: 510 (128/126/128/128)

Kaplan
FL 1 Beginning of studying: 498 (123/127/123/125)
FL 2 Mid-studying: 503 (126/126/126/125)
FL 3 May: 506 (127/126/127/126)

The Princeton Review
FL 1 5/3: 502 (124/126/125/127)
FL 2 5/7: 500 (125/125/124/126)

AAMC
Sample 5/20: (85%/87%/80%/81%)
Practice 5/26: 513 (129/128/128/128)

5) Major
Sociology

6) Tips
Don't void unless you have a reason - I seriously considered voiding my score and spent the whole month wishing I had. I am very happy with my score and grateful that I promised a friend I would not void under any circumstances (and kept my promise).

Ask all the questions you may have - ask a friend, a professor, a tutor, a forum, anyone! You won't know the answer unless you ask.

Take every practice test seriously and don't become discouraged if you do poorly; remember that they are very harshly scored.

7) Time Spent Studying
Four months total.
Three months of studying whenever I got a chance, one month of studying all day almost every day.
 
Last edited:
  • Like
Reactions: 7 users
1) Actual Score
516 (130/128/128/130)

2) Study Method

Four months before my exam, I began going through the Kaplan 7-Subject book set and reading each book cover-to-cover, taking notes as I went. I never actually used these notes, but summarizing what I was reading helped keep me engaged. I also did an occasional practice test. During this time, I live-streamed a few ThinkMCAT classes. I was also going to school full time, working, and volunteering.

A month before the exam, the semester was mostly over and I was able to just focus on studying. At this point, I had finished the content review and began focusing on the AAMC and Khan Academy materials and doing practice exams. I also made study sheets and had a friend quiz me on the material.

A week before the exam, I memorized everything in the Kaplan QuickSheet. I also watched more Khan Academy videos of concepts I was still just not understanding.

3) Materials (in order of usefulness)

Kaplan 7-Subject Review
This was my primary study tool. I used it to familiarize myself with the content.

Khan Academy Videos
Definitely important, but make sure you check the comments after watching a video because there are often mistakes.

AAMC Materials
I didn't find them as useful as many people say, but I used the practice questions and book until I felt that I had an idea of what to expect.

ThinkMCAT Classes (livestreamed)
For subjects you aren't comfortable with, this class will help you learn what is most important.

Princeton Review Materials
Used these books mostly for bio/biochem to cross-check what I should memorize, since Kaplan doesn't emphasize that.

4) Practice Tests (in no particular order)

Actual 6/2: 516 (130/128/128/130)

Next Step
FL 1 5/10: 510 (128/126/127/129)
FL 2 5/18: 510 (128/126/128/128)

Kaplan
FL 1 Beginning of studying: 498 (123/127/123/125)
FL 2 Mid-studying: 503 (126/126/126/125)
FL 3 May: 506 (127/126/127/126)

The Princeton Review
FL 1 5/3: 502 (124/126/125/127)
FL 2 5/7: 500 (125/125/124/126)

AAMC
Sample 5/20: (85%/87%/80%/81%)
Practice 5/26: 513 (129/128/128/128)

5) Major
Sociology

6) Tips
Don't void unless you have a reason - I seriously considered voiding my score and spent the whole month wishing I had. I am very happy with my score and grateful that I promised a friend I would not void under any circumstances (and kept my promise).

Ask all the questions you may have - ask a friend, a professor, a tutor, a forum, anyone! You won't know the answer unless you ask.

Take every practice test seriously and don't become discouraged if you do poorly; remember that they are very harshly scored.

7) Time Spent Studying
Four months total.
Three months of studying whenever I got a change, one month of studying all day almost every day.
Hey! I am so happy for you! You inspire me! Congratulations!
 
1) Your individual scores and composite score
Total 518 (97%) CP 130 (97%), CARS 128 (90%), BB 131 (99%), and PS 129 (92%)

2) The study method used for each section
CARS: I didn't study for it at all. I am a non-traditional out of college for more 10 years. As a lawyer, I read a lot. Surprised with the score, expecting to get something around 130. Oh well.
CP and BB. Intensive content review for 3 weeks, read old textbooks and Kaplan 7-book series back and forth until I literally memorized EVERYTHING. Then I started doing question packs, which I found very easy in comparison to the real test. Finally I focused on the section banks which I found quite challenging but representative of the real exam. As a second round of content review, I watched all Khan videos.
PS. Kaplan book memorized within 2 weeks, then moved on to textbooks (found online) and went through the glossary to make sure I didn't miss any. Did the PS section bank and went through every single term I didn't recognize. Made a short list to supplement the glossary.

3) What materials you used for each section(Kaplan, TPR, Examkrackers, AAMC, TBR, etc)
Kaplan 7-book Series is my main material. Rented EK from amazon and liked the illustrations but didn't find the content review detailed enough. Khan is a great resource for content review, but it can go overboard sometimes. Then I found old textbooks to plug holes where I felt the review books didn't give a good rundown.

4) Which practice tests did you use? (Optional: include scores)
Question packs Chemistry 91%, Physics 92%, Biology 1&2 92%, CARS 1 & 2 95%.
Section Bank, CP 74%, BB 71%, PS 72%
EK 1 CP 72%, CARS 89%, BB 75%, PS 75%
AAMC OG (30 questions) CP 92%, CARS 95%, BB 92%, PS 94%
Didn't have time to take any AAMC full length.

5) What was your undergraduate major?
Applied Mathematics

6) Any other tips you may have for those of us who still have this test lurking over us?
Memorize everything in the Kaplan 7-book series and understand everything. If not, do a google search and read upon it. Understand genetics and amino acid short hand notations. I realize that once I read the notation rules, all the biochem passages become very accessible.

7) How long did you study for the MCAT?
Full-time study for 1 and half months (8-10 hours a day). No weekend break.
 
  • Like
Reactions: 1 user
Finally I get to post in this thread. Fresh out the oven my score was released today~

1) Your individual scores and composite score

1st MCAT: 26 (PS 9/VR 7/BS 10):oops: taken Jan 2015
2nd MCAT: 515 (CP 130/CARS 125/BB 130/PS 130) taken Jun 18, 2016

2) The study method used for each section
CP and BB: I have equations written on sticky notes in front of my desk so I stare at them all the time until I memorize it. I also have an amino acid sheet with the structures and everything on my desk. I have a decent background in these sections so I just did passages and reviewed them. Really know your equations though. I've read a lot of research papers during my undergrad years and that definitely helped with analyzing graphs and research based passages. I was also really familiar with the lab techniques because I did a lot of them myself. For passages that involve new pathways, I suggest you draw them on your scratch paper as you read them so you can refer to your diagram when you do the questions instead of going through the entire passage. I think a lot of the higher level classes in my major helped as well (e.g. immuno, microbio, cell bio). Read lots of research papers.

CARS: Okay, I'm ESL so I don't have superb advice for CARS but if you're also ESL I hope my words give you some idea on how to improve. Like many ppl on this forum have said before you can't bring in your own opinions for this section. Try to minimize your reactions and read objectively. All the answers you need are in the passage. It's really about what the author thinks. Also DO NOT REREAD. After I started doing that, I realized my score for CARS didn't drop but I actually had more time to review which was extremely helpful during my test day. This was because each time I reread, not only was I losing time on easy questions in later passages but also I couldn't find the answer to the questions I had difficulty on. And what were the questions I had difficulty on? The ones that didn't have direct answers in the passage so no matter how many times I reread I would never find the answer. So just do yourself a favor and don't reread. However, do try to analyze the author's main point as you read. Think about the connection between each paragraph and the author's argument. Is it supporting the argument? Or is it a counterexample of an argument presented by another person (which means that's totally not the author's opinion)?
This thread was really helpful: http://forums.studentdoctor.net/threads/testing-solution’s-30-day-guide-to-mcat-cars-success.1131850/#post-16422962
Another thing I did between my first exam and retake was I translated a lot of fanfic (don't make fun of it, it actually worked for me, fanfic with good writing though). It really required me to look up a lot of terms that I'd normally ignore when I'm just reading casually. Although my CARS score ain't stellar, I did feel I improved my English a lot through translation work. I also did some video subbing and web comic translations. Any type of translation work would help IMO if you're ESL.

PS: I never took sociology during undergrad so I studied that part on my own with the Kaplan book. I basically wrote down all terms I've never seen before and reviewed them daily. Inequality is a topic that appears a lot. Also get your hands on the 100 page Reddit Khan Psych notes. Good stuff for general daily review. (Kudos to the author of that pdf document! I'm sure many sdners have used it to study PS)

3) What materials you used for each section(Kaplan, TPR, Examkrackers, AAMC, TBR, etc)
I used the Kaplan books for biochem and behavioral sciences. I had TBR for chem, orgo, physics and bio when I was studying for the old exam. I also used EK 1001 questions but it wasn't that helpful because I thought the questions were a bit too easy. Only use it if you're still reviewing concepts. I used TPRH and EK 101 (don't do the later passages for EK the questions and passages suck) for CARS. Everything AAMC offered. Google is your best friend for PS. I also made mini tests by combining passages from each section. I take them timed and I grade them at the end.

4) Which practice tests did you use? (Optional: include scores)
- All the old AAMC exams
- AAMC unscored: 73%/72%/75%/80%
- AAMC scored: 511 (129/126/128/128)

5) What was your undergraduate major?
Molecular Bio

6) Any other tips you may have for those of us who still have this test lurking over us?
I really rushed to take my 1st MCAT since it was the last exam in Jan 2015 and that was a bad idea. The format was soooo different than the old AAMC practice exams and more like the current one. I also thought I bombed CP and PS for my 2nd exam (I blanked out for the 1st CP passage the first question had me like ??? and PS was WTF are these random terms I've never seen them before???). So don't let post test feelings get to you, they're mostly inaccurate! I'm just glad my CARS is minimum lol
Also being a 2 time taker I can tell you that the real deal is always harder than the AAMC FLs. The real thing will definitely include harder passages similar to the AAMC section banks and new terms you've never seen before. Don't panic, if you can't figure it out within the next minute just mark it and move on. I had leftover time for all my sections, even for CARS I had 10min left. If you haven't encountered those terms before other ppl are probs in the same boat. Also I get that for FLs you sometimes won't have time to review all marked questions. What you want to do is maximize your score so I divide marked questions into 2 categories: 1) ones I have no clue on I just guessed 2) I have some clue on it, or it's a calculation I probs need to double check and if I check it I'll get it right for sure. For the 2nd type I mark it and write the number down on my scratch paper and I check those 1st. It's more likely I turn a wrong answer into a right one in this case because it's easier to spot a stupid mistake than trying to guess sth random. I do this for all exams in general. Prioritize the questions you need to check. You want to make sure you get all the questions you know correct and avoid stupid mistakes, only guess on those you have no idea on.

Overall advice: ONLY TAKE THE MCAT WHEN YOU ARE READY

I was really nervous for my 2nd exam I even had my brother check my scores for me lol
Good luck to everyone I hope this helps. I literally owe my score to this forum.

7) How long did you study for the MCAT?
1st MCAT: 1.5-2 months
2nd MCAT: 1 month hardcore - lol don't do this I was studying 12-13 hours a day w/o breaks it hurts your brain. I pretty much only studied for biochem, PS, and CARS. I knew my strengths and weaknesses pretty well plus I took the MCAT before so I knew what to expect.
 
Last edited:
  • Like
Reactions: 4 users
1) Your individual scores and composite score
519 (CP 132/CARS 126/BB 132/PS 129)

2) The study method used for each section
CP: I'm a chemistry TA, so I barely reviewed chem at all. I hadn't taken physics for 2-3 years, so I did extensive content review and memorized all of the formulas. Even though I memorized every formula, I do not remember using a single one on the actual MCAT. Most of the questions were theory and concept based.
CARS: It's obvious that my CARS score is lacking compared to my other subsections, and if I could go back and practice more, I would. It was always my weakest section, but I tried to salvage it too late in my preparation (in my final two weeks of preparation). I would recommend doing one practice passage every day a few months before your exam date. Contrary to what many people say, I found that I did better if I took the time to understand the passage extremely well on my first read through and take the questions faster. Try a few different approaches and find what works for you.
BB: Heavy content review because there's just so much content. This is a frustrating section, because you study everything and see 5% of the content you studied or less on the actual exam. I did more practice passages on this section than any other, and it paid off. Khan Academy videos and passages were invaluable here, and I would highly recommend doing them, especially because they're FREE!
P/S: I took social psychology the semester before the MCAT, which was the most beneficial class ever for this section. It taught me 50% of the material. Other than that, I did light content review and barely touched it.

3) What materials you used for each section(Kaplan, TPR, Examkrackers, AAMC, TBR, etc)
C/P: TPR content review, AAMC Section Bank, Khan Academy videos for select physics concepts
CARS: TPR, NextStep
B/B: TPR content review, AAMC Section Bank, a few Khan Acacdemy videos, ALL of the Khan Academy practice passages in final 2 weeks
P/S: TPR content review, AAMC Section Bank, a few Khan Academy videos, ALL of the Khan Acacdemy practice passages in final 2 weeks

4) Which practice tests did you use? (Optional: include scores)
(in this order)
TPR 1: 503
TPR 2: 507
Next Step free practice exam: 512
TPR 3: 504
AAMC Scored: 513
AAMC Unscored: 85.5%

5) What was your undergraduate major?
Health Science

6) Any other tips you may have for those of us who still have this test lurking over us?
Don't just study hard, study smart. Find the areas in which you're not doing well, and study them until you understand them. Focus on concepts rather than specifics. Most importantly, and perhaps quite underrated, eat well, sleep well, and exercise during your prep. Start going to bed early and waking up early a couple weeks before your exam so you can go to bed early and feel refreshed the morning of. I even took a short run to get myself going that morning. If you can, keep a solid support system that can motivate you when you're feeling burnt out and talk you down when you're feeling unreasonably stressed. We've all been there. And hopefully, when you see that score, you'll feel the utmost relief that it's all over (but really just beginning because applications will be upon you!).

My other, more specific advice is to use AGGRESSIVE process of elimination! I guarantee that this is how I got 132s in C/P and B/B, because I remember making educated guesses on a fair amount. Also, don't take how you felt about the exam too seriously. I thought I bombed C/P and rocked B/B, and ending up doing great in both. Take all of your practice exams as if it's the real deal: timed, full effort, all in one sitting.

7) How long did you study for the MCAT?
A little less than 2 months full time, 6-10 hours/day.

GOOD LUCK TO ALL!!
message me if you have any further questions or just want to chat!
 
  • Like
Reactions: 4 users
1) Your individual scores and composite score
Overall: 518
CP 129, CARS 132, BB 130, PS 127
2) The study method used for each section
I read through the review books and complemented this by reading through old notes from classes, if I had the notes on that specific topic. If I still didn't understand a specific topic, I'd watch a Khan Academy video on it. For CARS, I strictly did practice passages. I used the AAMC Section Bank problems as well as the Khan Academy practice questions.
3) What materials you used for each section(Kaplan, TPR, Examkrackers, AAMC, TBR, etc)
I used the Kaplan review books for the chemistry and biology/biochemistry sections. I did not read the chapter introductions/conclusions. I did not do the section questions or the review questions at the end of each chapter. I used the ExamKrackers for the Psychology/Sociology sections. I also used Khan Academy videos throughout for topics that needed extra reinforcement.
4) Which practice tests did you use? (Optional: include scores)
I did the non-scored AAMC practice. Also, about a week before I took the MCAT, I took the first scored AAMC Practice Test, in which I scored a 516.
5) What was your undergraduate major?
Biology
6) Any other tips you may have for those of us who still have this test lurking over us?
I found that brute memorization didn't help me as much as doing practice problems. Yes, there're a lot of things you have to memorize, such as amino acids, metabolism, etc... However, you'll be much more successful if you can employ critical thinking skills and strong reasoning. The only behavioral class that I had taken coming into the test was Gen Psych. I also had not taken second semester physics or a physiology class. I didn't even bother trying to learn the second semester physics stuff, except for Ohm's Law. Don't overthink your studying method too much. You know how you learn best, and you know how much time you need to invest in order to learn the material.
Also, during the actual MCAT, I recommend taking all the breaks, so you can give your mind a break. Pack a good lunch, take a deep breath.
7) How long did you study for the MCAT?
5 weeks. I studied every day over winter break and took the test at the end of January
 
  • Like
Reactions: 1 user
1) Your individual scores and composite score
Total: 521 PS: 130 CARS: 129 BIO:130 PSY:132
2) The study method used for each section

I took Human Physiology, Cell Biology, Psychology, and Organic Chemistry II+lab last semester and took the MCAT the week after finals. About a month before the exam, I took 5 or so Full-Length exams from Kaplan, and averages 504. I also took the unscored AAMC exam.
3) What materials you used for each section(Kaplan, TPR, Examkrackers, AAMC, TBR, etc)
I bought the 7-book Kaplan set but didn't really use it. I used 5 Full-length exams online.
4) Which practice tests did you use? (Optional: include scores)
5 Kaplan Full-lengths and AAMC unscored (averaged 85% on sections).
5) What was your undergraduate major?
Biochemistry
6) Any other tips you may have for those of us who still have this test lurking over us?
Taking classes over a semester really help you learn, not just memorize, the information. I only took the middle break during the exam, but not the other two.
7) How long did you study for the MCAT?
1 semester unofficially (4 weeks officially).

I know it doesn't happen the same way for everyone (this is a miracle to me). I still thank God every day for this. Please don't wait till the last minute to start studying (or decide to be pre-med for that matter)!
 
Last edited by a moderator:
  • Like
Reactions: 1 users
1) Your individual scores and composite score
Total: 521 PS: 130 CARS: 129 BIO:130 PSY:132
2) The study method used for each section

I took Human Physiology, Cell Biology, Psychology, and Organic Chemistry II+lab last semester and took the MCAT the week after finals. About a month before the exam, I took 5 or so Full-Length exams from Kaplan, and averages 504. I also took the unscored AAMC exam.
3) What materials you used for each section(Kaplan, TPR, Examkrackers, AAMC, TBR, etc)
I bought the 7-book Kaplan set but didn't really use it. I used 5 Full-length exams online.
4) Which practice tests did you use? (Optional: include scores)
5 Kaplan Full-lengths and AAMC unscored (averaged 85% on sections).
5) What was your undergraduate major?
Biochemistry
6) Any other tips you may have for those of us who still have this test lurking over us?
Taking classes over a semester really help you learn, not just memorize, the information. I only took the middle break during the exam, but not the other two.
7) How long did you study for the MCAT?
1 semester unofficially (4 weeks officially).

Hi O.T.I.! I;'m actually planning on studying for the MCAT this upcoming fall semester (taking MCAT in January), during which I'll be taking Biochemistry, Human Physiology, and Physics I. My biggest question is, how did you manage to balance studying for the MCAT topics outside of your classes with all your coursework? In total, I'll be taking about 16 credit hours+research, so I'm not really sure how to balance everything.
 
Hi O.T.I.! I;'m actually planning on studying for the MCAT this upcoming fall semester (taking MCAT in January), during which I'll be taking Biochemistry, Human Physiology, and Physics I. My biggest question is, how did you manage to balance studying for the MCAT topics outside of your classes with all your coursework? In total, I'll be taking about 16 credit hours+research, so I'm not really sure how to balance everything.
Honestly, not very well. Taking h. Phys, biochem and psych meant I didn't cracking a book open on the bio/biochem or psych/soc section. For you it will be chem/phys. So at around April (so for you ca. October) I started using my Fridays and/or Saturdays to take an FL and READ THE RATIONALE for every question. To me, that was the most effective way of learning new info (reading a book of stuff just didn't sit well with me). When I came across a rationale with a topic I was unfamiliar with, I read up on THAT. I also took more section tests in the areas I had to brush up on (so for me phys/chem). I didn't do much for CARS, because that wasn't a problem for me (lots of timed writes in high school and scientific papers in college research).

May God help you succeed even better than I did! Feel free to ask any more questions!
 
  • Like
Reactions: 1 users
1) Your individual scores and composite score
First Attempt:
Chem/Physics: 125
CARS: 127
Bio: 127
Psych/Soc: 129
Composite: 508

Second Attempt:
Chem/Physics: 126
CARS: 132
Bio: 125
Psych/Soc: 126
Composite: 509
2) The study method used for each section
Each section was a combination of content review and practice questions and exams. It was all self-directed, no classes or private tutors. Both attempts, I gave myself about 10-12 weeks to prepare.
3) What materials you used for each section(Kaplan, TPR, Examkrackers, AAMC, TBR, etc)
I tried TPR and hated it. I felt like it was a waste of time since it covered such broad content with the hopes that it might appear on the test. It did not feel like it was relevant. For my first attempt, I used Khan Academy heavily -- watched nearly every video, especially for the science sections and did lots of practice passages. It decently prepared me, but the practice passages weren't similar enough to the real exam. The second attempt, I exclusively used Examkrackers. I loved their books and would highly recommend that you don't waste your time with TPR (I can't speak to Kaplan or other programs). EK does an excellent job of covering relevant content and providing highly realistic practice passages and exams. I had poor timing on the Chem/Physics sections on both attempts, usually rushing on the last 7 or so questions. I feel like if I were faster, my scores would have been much better, because EK did prepare me well to know the content.
Also, EK did an excellent job of preparing me for the CARS section. I was shocked when on my first attempt, I only got a 127 in CARS. On every practice exam, I had always gotten a perfect score in that section. But, I had taken it for granted and doing the occasional TPR passage didn't prepare me enough for what turned out to be a challenging CARS section on the real thing. For my second attempt, I did everything that EK told me to do, and clearly it worked.
The only thing that I feel no one does well yet is preparing students for the Psych section. The first time I took it, I felt terrible about the section and was shocked by the 129. The second time, I still felt terrible, and didn't do as well. There is still content the MCAT uses that the prep companies have not picked up on yet and it really feels like a guessing game most of the time. This is coming from someone whose major mandated psychology, communication, and sociology classes!
4) Which practice tests did you use? (Optional: include scores)
I used EK tests 1-3 (got tired of paying $50!) as well as both official AAMC tests. EK does those little half tests at the end of each chapter -- those were amazingly useful.
5) What was your undergraduate major?
Economics B.S.
6) Any other tips you may have for those of us who still have this test lurking over us?
Perhaps this is better in the "should I retake" thread, but damn, how I wish I would have just gotten my perfect CARS score the first time! I would have had a 514. Be careful and try to get it right the first time. I was so anxious about retaking because I knew that dropping a point or two was possible. Thankfully I netted one extra point, but it really could have gone the other way. Also, don't waste your time with TPR. Just use EK. If I would have used EK the first time, I think I would have gotten the score I wanted and would not have felt obligated to do the retake. Timing is crucial. Things will not go perfectly on test day (my Citrix connection was disrupted in the middle of the Chem/Physics section on my second attempt and the time continued to run even though I was locked out of the test for a few minutes). Be fast enough that losing a few minutes won't affect your ability to complete the test, like it did with me. Also, as I've been reminded by doctors and advisers and nice people, the MCAT is just ONE part of the application. I was told directly by the admissions dean for a top school that if you're over the 80th percentile, you'll be just fine. If you're on this thread shooting for the 509+, don't stress if you do "only" get a 509 or a bit lower. The average accepted score last cycle was a 506 (my premed adviser just told me that earlier today). Just do your best :)

7) How long did you study for the MCAT?
About 10-12 weeks. The first attempt, I studied over my summer break. The second attempt, I had already graduated and was working full-time. Studying was crammed to the late evenings and weekends. Not ideal (certainly doable) but if you have the luxury of a summer break, take advantage of it.

Do you mind sharing your EK exam scores? And also for the second attempt did you retake any aamc material/was it indicative the second time around
 
Members don't see this ad :)
1) 521 total 130 PC/129 CARS/130 Bio/132 Psy/Soc
2)Explicitly used Kaplan's materials and study guide for each section
3)I used only Kaplan and aamc material. Kaplan for content studying. Kaplan Q-bank and aamc Q packs for daily practice. Kaplan FLs 1-7. And AAMC FL right before exam. Also did the 120 Qs in the official guide to the MCAT.
4)Kaplan FLs 1-7 scoring ranged from 501-508. AAMC practice right before exam ~86%
5)Biology @ top 100 state school
6)I did every single little thing my Kaplan Course had to offer. I swear by them, they helped me so much. Which is why I now teach for them:) I strongly suggest taking a course. When you do, do every single reading, watch every single video, every practice Q they got. Suck everything you can out of that course.

My schedule was this. I followed their instructions to a Tee, except didn't just do "recommended" material, I did everything the course had to offer. around 5 hours a day. When the course was over, I still had 8 weeks till my exam. So I took a FL every weekend, and spent the rest of the week doing focused content on what I got wrong. And doing practice specific to that area. Thats it folks. I simply followed Kaplan, and did FLs after the course ended.

One thing I think REALLY heped.... do one practice passage from each of the 4 section EVERY DAY (except test days). I used the aamc Qpacks and official guide, and the Kaplan Q bank for these practice passages. Don't skip this ever. Do them. Do them Do them Do them. Oh one more thing. DO THEM. one P/C, one CARS, one BIO, and one Psy/soc every day.

5) 5 months for at least 5 hours a day...... yes folks... thats 5 x 30 x 5=750 hours... its actually more than that because FLs were 7 hours long.

Note: I'm not that bright... AT ALL. I do have a 3.99gpa, but thats only because I work myself to the bone!!!! Most people won't have to put in nearly 750 hours to do this well... probably around 300-350 from what I can see. BUT... if you're dumb like me... DON"T BE DISCOURAGED!!!! JUST WORK YOUR @$$ off. You can do it! WHO CARES IF THEY SAY 300-350 hours to prepare???!!! TRY TO DOUBLE THAT IF YOU KNOW YOUR NOT SMART!! I guarantee you can out work people smarter than you, and grab a disgusting score.

Good luck
Sooo me, with the whole working out people smarter than me by just studying way extra. Sometimes I wonder how they do it, just cram something a day before and kill an A. But yess glad theres more like me out there
 
  • Like
Reactions: 1 users
1) Your individual scores and composite score
Total: 521 PS: 130 CARS: 129 BIO:130 PSY:132
2) The study method used for each section

I took Human Physiology, Cell Biology, Psychology, and Organic Chemistry II+lab last semester and took the MCAT the week after finals. About a month before the exam, I took 5 or so Full-Length exams from Kaplan, and averages 504. I also took the unscored AAMC exam.
3) What materials you used for each section(Kaplan, TPR, Examkrackers, AAMC, TBR, etc)
I bought the 7-book Kaplan set but didn't really use it. I used 5 Full-length exams online.
4) Which practice tests did you use? (Optional: include scores)
5 Kaplan Full-lengths and AAMC unscored (averaged 85% on sections).
5) What was your undergraduate major?
Biochemistry
6) Any other tips you may have for those of us who still have this test lurking over us?
Taking classes over a semester really help you learn, not just memorize, the information. I only took the middle break during the exam, but not the other two.
7) How long did you study for the MCAT?
1 semester unofficially (4 weeks officially).
Yo We're MCAT twinsies. Our overall and sub scores are identical
 
  • Like
Reactions: 1 users
1) Your individual scores and composite score

509 126/125/130/128

2) The study method used for each section

P/C: lots of KA videos and practice sections with thorough review. Section Banks were most helpful. I kept a notebook with notes for each question I got wrong. Obviously not my strongest section, but I was satisfied with my score. My physics and Chem background is terrible so I pretty much had to relearn everything. Definitely memorize amino acids.

CARS: usually my strongest section. I was scoring 129+ on practice tests so I didn't do much prep for it besides FLs. Not sure what happened, but the actual test just felt longer and more convoluted than anything I had experienced. Maybe it was because I was so nervous and hyped up on caffeine. Regardless, totally bummed when I saw this score. My best advice for this section is to practice timing and identifying question/answer types.

B/B: I was very pleasantly surprised with this score. I was averaging 125/126 on practice tests. The last few weeks I really focused in on the section banks and understanding why I was getting questions wrong. I started to realize that SO much unnecessary info was included in passages. Understanding the overall gist of the passage, including accompanying data (if applicable) was the most important aspect. I felt like I had a lot of pseudo-discretes on my exam. Memorizing amino acids is definitely useful for this section too. I also had a few questions related to cellular respiration so I was glad I reviewed that shortly before my test.

P/S: scored 128+ on all practice tests. I usually loved this section but again felt as though it www totally different than practice material I had encountered. There was A TON of sociology on my test, which I was not prepared for. Not gonna complain about a 128 though, as I left feeling like I blew it.

3) What materials you used for each section(Kaplan, TPR, Examkrackers, AAMC, TBR, etc)

I did a Kaplan course a while back but honestly didn't put as much effort into it as I should have. I think I took it too early. I read sections from Kaplan books occasionally but I liked KA videos better and they seemed more applicable to the test. Kaplan books seemed to have a lot of unnecessary info. I also bought and used all the AAMC materials. I focused on them the month before the test. I ended up getting a puppy during that time (not recommended) so I didn't go as hard as I wanted to leading up to the test. AAMC materials are definitely the most important in my opinion, especially section banks and the FLs. I will say though, my test felt much harder to me than either AAMC practice test.

4) Which practice tests did you use? (Optional: include scores)

Kaplan: first test was a 496, last test was a 504. Took 7 total.

AAMC:
Unscored: ~75% (60%/92%/60%/89%)
Scored: 509 (124/130/126/129)

5) What was your undergraduate major?

Neuroscience

6) Any other tips you may have for those of us who still have this test lurking over us?

Give yourself plenty of time to do AAMC materials and review them thoroughly. Take practice tests under testing conditions. My first few practice tests were in my apartment, but then I started making myself wake up early and drive to my school to take them. Also, try to get on a good sleeping schedule a few weeks before your test. I'm a total night owl and not a morning person so I knew it would be important for me to train myself to go to bed/wake up early.

7) How long did you study for the MCAT?

On and off for about 4.5 months. I only really buckled down the last couple of months. Wish I would've been a little more disciplined though to be honest.




Sent from my iPhone using SDN mobile
 
1) Your individual scores and composite score
Total: 522 (C/P: 131 CARS: 132 B/B: 129 P/S: 130)

2) The study method used for each section
C/P and B/B, read EK (9th edition) cover to cover, and took my own notes, compressing down the material into what I needed to know. Reviewed my prior notes every night. CARS I only used practice passages. Watched Khan Academy for P/S and made my own Anki deck, reviewing that daily.
Did science content review, reading and rereading EK (googling topics if I needed more information) for about two weeks.
After two weeks, I switched to practice problems (AAMC q packs/section banks) and then full-lengths for the next two and a half to three weeks.
I was always above 90% on AAMC CARS (reading is my strength), so the only practice was the q packs and full-lengths.
I would do a full-length, take the rest of the day off, and then spend the next day or two (depending on when my next FL was scheduled) going over every question on the test. If I got the question right, I would think about if I knew the content or if it were a guess. If I got it wrong, I would categorize it as either a content knowledge or just generally not reading the question carefully.

3) What materials you used for each section(Kaplan, TPR, Examkrackers, AAMC, TBR, etc)
Used EK (9th edition) for the science sections content. Just read, did not do any of their questions. Khan Academy for P/S and the accompanying notes.
AAMC question packs / section banks helped immensely.
NS full length exams.

4) Which practice tests did you use? (Optional: include scores)
I used the NS practice exams and the AAMC exams. 515 on NS 1 and 2, and 512 on NS 3 and 4.
521 on the AAMC Scored (taken three days before)
~88% total on the AAMC Unscored (taken one week before)

I thought the NS exams were good, albeit difficult, especially the science sections. The CARS felt very hit or miss and very weird, so I always took my score on that section with a grain of salt. The P/S section was sometimes WTF-inducing, but that's pretty representative of the actual exam. The good thing is that NS gives you the percentage of people who chose a certain answer, so you can see if some of the questions are kind of BS. I would recommend the NS exams. However, I don't have any other third party exams to compare them to.

During my practice tests, I simulated the environment of the actual test as much as I could. Had a spreadsheet that documented the time I woke up, what I ate, how much coffee I drank (a lot), the time I started the test, how I felt, etc.
While I totally understand that this is neurotic, I think a large part of the prep for this test is getting yourself used to taking the test and building up the necessary stamina. It was an invaluable element in my studying.

5) What was your undergraduate major?
English literature, baby! (Had just finished a post-bacc with all of my prereqs except O Chem II, though.)

6) Any other tips you may have for those of us who still have this test lurking over us?

I think a positive mindset goes a long way. I was absolutely convinced I was going to do poorly a lot of the time while studying, but during the actual test, I just tried to trust my studying and went into the test feeling very confident.

Also, your post-actual exam feelings mean absolutely nothing. My C/P section was brutal. I blindly guessed on about 5 questions (calculations I didn't feel like doing or concepts I just didn't know), and marked at least 20. Was convinced I would be lucky to get a 125 on that section. Turns out I got a 131.

Don't look up answers/questions you remember after the test. I did this and overwhelmingly found that I had answered wrong (because it's always the tough questions that get stuck in your head), and it was discouraging.

If you're studying full-time, give yourself breaks. Sometimes I would wake up and not feel like studying at all, or just would feel tired. So I'd let myself sleep, or I'd take the morning off and go for a hike. Watch your body/mental state. This allowed for me to make my studying of a high quality and very focused (e.g. 4 hours of book time vs. 9 hours of books/facebook/texting).

Brought eye drops and caffeine pills for my breaks. Very helpful.

7) How long did you study for the MCAT?
5 weeks full-time (~2 weeks of science content review and then ~3 weeks of AAMC practice problems or FLs and reviewing). Created an Anki deck for P/S and started reviewing P/S terms about 4 months before the test, though.
Probably a total of ~200-250 hours, though I recognize had a solid base, seeing as how my prereqs were fresh in my mind, and I didn't really do much CARS practice, which would have added many more hours.

Happy to answer any other questions via PM if anyone would like.
 
Last edited:
  • Like
Reactions: 4 users
1) Your individual scores and composite score

509 126/125/130/128

2) The study method used for each section

P/C: lots of KA videos and practice sections with thorough review. Section Banks were most helpful. I kept a notebook with notes for each question I got wrong. Obviously not my strongest section, but I was satisfied with my score. My physics and Chem background is terrible so I pretty much had to relearn everything. Definitely memorize amino acids.

CARS: usually my strongest section. I was scoring 129+ on practice tests so I didn't do much prep for it besides FLs. Not sure what happened, but the actual test just felt longer and more convoluted than anything I had experienced. Maybe it was because I was so nervous and hyped up on caffeine. Regardless, totally bummed when I saw this score. My best advice for this section is to practice timing and identifying question/answer types.

B/B: I was very pleasantly surprised with this score. I was averaging 125/126 on practice tests. The last few weeks I really focused in on the section banks and understanding why I was getting questions wrong. I started to realize that SO much unnecessary info was included in passages. Understanding the overall gist of the passage, including accompanying data (if applicable) was the most important aspect. I felt like I had a lot of pseudo-discretes on my exam. Memorizing amino acids is definitely useful for this section too. I also had a few questions related to cellular respiration so I was glad I reviewed that shortly before my test.

P/S: scored 128+ on all practice tests. I usually loved this section but again felt as though it www totally different than practice material I had encountered. There was A TON of sociology on my test, which I was not prepared for. Not gonna complain about a 128 though, as I left feeling like I blew it.

3) What materials you used for each section(Kaplan, TPR, Examkrackers, AAMC, TBR, etc)

I did a Kaplan course a while back but honestly didn't put as much effort into it as I should have. I think I took it too early. I read sections from Kaplan books occasionally but I liked KA videos better and they seemed more applicable to the test. Kaplan books seemed to have a lot of unnecessary info. I also bought and used all the AAMC materials. I focused on them the month before the test. I ended up getting a puppy during that time (not recommended) so I didn't go as hard as I wanted to leading up to the test. AAMC materials are definitely the most important in my opinion, especially section banks and the FLs. I will say though, my test felt much harder to me than either AAMC practice test.

4) Which practice tests did you use? (Optional: include scores)

Kaplan: first test was a 496, last test was a 504. Took 7 total.

AAMC:
Unscored: ~75% (60%/92%/60%/89%)
Scored: 509 (124/130/126/129)

5) What was your undergraduate major?

Neuroscience

6) Any other tips you may have for those of us who still have this test lurking over us?

Give yourself plenty of time to do AAMC materials and review them thoroughly. Take practice tests under testing conditions. My first few practice tests were in my apartment, but then I started making myself wake up early and drive to my school to take them. Also, try to get on a good sleeping schedule a few weeks before your test. I'm a total night owl and not a morning person so I knew it would be important for me to train myself to go to bed/wake up early.

7) How long did you study for the MCAT?

On and off for about 4.5 months. I only really buckled down the last couple of months. Wish I would've been a little more disciplined though to be honest.




Sent from my iPhone using SDN mobile

Just wanted to ask you what was your study schedule. Approx. Hrs per day, days per week. And if you can describe your everyday schedule. I.e. day 1- TBR content review, passages etc. (Don't list every day. Just a sample day). I'm trying to figure my schedule and have been reading on different approaching here in SdN. Do you recommend all content review first, then practice passages, tests etc.?
 
Based off prelims. I'll update when final scores come out in a week but it should be in line.

  • C/P : 74-89 (127) ... Princeton Review - Took mad notes. Made notecards to memorize formulas. Practiced with Khan passages.
  • CARS : 76-91 (127) ... Nothing. The practice tests I took were all the studying I did. Probably should've done more but Chem & Physics were much more pressing issues during my study.
  • Bio : 72-87 (127) ... Undergrad classes, slight TPR review, & youtube <3. Made a few notecards.
  • Psych : 85-100 (129-132) ... Princeton Review.

TOTAL: 510-514 (likely 510 or 511 but who knows)

Did three TPR practice tests (496 - 498 - 501) and the AAMC FL (57%, 87%, 73%, 75%).

Undergrad major was Biology with Chemistry minor.

6) Any other tips you may have for those of us who still have this test lurking over us? Yeah, DONT GET DISCOURAGED. On my first practice test a month & a half out, I got a 496. I wanted to cry. I was demoralized. Somehow it just made me angry. Then I studied like crazy for a month or so and did much better than I ever thought I'd do. Keep at it. Channel all energy into murdering the test.

7) How long did you study for the MCAT? 1 month.

Also, the best classes for this test are microbio, genetics, and biochem (obviously). Any upper level biology class that is research oriented will help immensely.

How critical is it to take microbio and genetics if you want to do well? I'm a neuro major so those classes aren't really requirements for me. My intro bio covered a little genetics/microbio though I believe. Not in depth like an actual course though
 
1) Score: 524 (99th Percentile); 132 Chem/Phys, 131 CARS, 132 Bio, 129 Psych/Soc
2) Study Method: Content review by prep books for the first two months, (roughly one chapter per day) including all of the chapter questions. Last month: reading through notes and then taking each of the 30 min unit tests provided and reviewing chapter questions from before. Last week: turning all of the important topics into flashcards and reviewing ad nauseum.
3) Materials: ExamKrackers and the AAMC Flashcards
4) Practice tests: Just the official aamc practice exam (514 - took two weeks before MCAT) and the unscored aamca practice exam (~520 - one week before MCAT). I had already taken the MCAT a year and a half ago, so I was pretty sure how good my baseline was (33 - 11/11/11).
5) Undergrad major: Double major, Molecular and Cellular Biology and Mesopotamian History.
6) Tips:

TEACH IT TO SOMEONE ELSE. For any subject you don't understand that well, or can't quite fully grasp, explain/teach it to someone else. It is 100% the best way to really understand a topic and be able to apply it on the test.
More than knowing the topics, the test really makes you use critical thinking and understanding of whole concepts and how they relate to systems as a whole. Teaching it to someone else makes it so much easier to apply what you know on the test.

7) How long: Little under three months. I was also working full time.

You mentioned teaching the subject to someone else really helps. I have a potential tutoring opportunity available.. would tutoring help in the way you mentioned or do you need some higher level analysis of the subject? I don't know I'm probably just overthinking this..my guess is yeah although it would be time consuming so I want to be 100% sure.
 
1) Your individual scores and composite score:
Total: 517 PS: 129 CARS: 129 BS: 130 Psych: 129

2) The study method used for each section
PS: HEAVY use of Khan Academy. This was my weakest area when I took the AAMC Official exam 2 week into studying. As I did for every section, I did content review and question right from the start. I probably watched more videos for this than any other section. Khan is a huge help for this section, as it's mostly conceptual.
CARS: PRACTICE PRACTICE PRACTICE. Seriously. Do at least 3 passages a day, 7 days a week. Use different materials. I used EK, TPR, Next Step, and AAMC materials. I probably did a full section once a week for the last two months of my studying. Some weeks I did two full sections. Review every single problem. Especially for the AAMC material.
BS: Lots of scientific reasoning here. Be sure you understand experimental methods. I did lots of flash cards for weak areas, and used Khan for some Biochem stuff, same as every section- lots of practice problems.
Psych: I definitely put the least effort into Psych. I did Khan videos when I was too tired to do anything else that required a lot of thought. Focus on passages that emphasize experiment design. Know the basic psych terms. Most likely when you take this on the real thing you're going to feel like you're failing... horribly. I don't normally like to say this in academics, but common sense helps a lot here.

3) What materials you used for each section(Kaplan, TPR, Examkrackers, AAMC, TBR, etc):
PS: Heavy use of Khan and EK. Khan mostly for videos and some practice. EK I used the 1001 books on top of their content review books.
CARS: Literally anything I could get my hands on. Especially AAMC material.
BS: Lots of Khan questions, to a lesser extent Khan videos. Most EK again. I did some NS question books (they're infuriating, though), and TPR questions.
Psych: Um... I guess I did Khan videos, I also did all of EK stuff. I used the AAMC official outline to "study." Anything that tests research design is good.

4) Which practice tests did you use? (Optional: include scores):
EK1, EK2, EK3: I scored around 70% on each. These are the best exams other than AAMC
AAMC FL: Two weeks into studying I took this, 64% Chem, 73% Bio, 85% CARS, and 64% Psych.. something to that effect. I retook it three days before the real deal and scored >90% on each section except PS which was high 80's.
TPR1, TPR2, TPR3: Scored 502 on each of these. Did poorly in PS each time, average BS and CARS. Psych was always high.
AAMC Official Guide Questions: 70% Bio, 80% CARS, 86% PS...I forget my Psych score. This test was hard compared to the official FL.
I also used ALL AAMC question packs. They come as packets of 120 questions. I did 60 questions at a time, timed. Treat these like a FL. I scored around 80% of CARS, 93% for Bio, ~85% for chem, ~75% for Physics,

5) What was your undergraduate major?
Biology. I have a degree in nursing as well.

6) Any other tips you may have for those of us who still have this test lurking over us?

Practice from the start. Used a mix of practice materials if you can afford it. Use Khan, it's free, it's good, and it's affiliated with AAMC. Use all AAMC material available. No excuses here.

Time everything.
EXPERIMENTAL DESIGN. It's on every section other than CARS. Know it. Be comfortable interpreting graphs. Be aware: AAMC likes to trick you on data interpretation. Know how to find a confounding variable, ect...

CARS every day.

Take notes when you read.

Read through your chapters at least twice. especially areas you're uncomfortable with
.
Make a word document called "lessons learned." Make notes on what you got wrong, how you can get to the right answer, and facts associated with that subject, and tips. review this at least once a week. By a month into study, you'll have a huge word doc. It helped keep me focused on my weak areas.

Don't study stuff you know more than once.

On the flip side, study stuff you don't know. I can't tell you how many times I googled redox reactions, electrolysis, galvanic cells.. etc. I hated it, but I was weak in it. You need to master the things you aren't comfortable with.

Practice> Review. Towards the end I wasn't reading any books except for specific questions. I used Khan videos, and google. Almost all my time was spent doing practice problems, timed practice problems. When you get a question wrong, guessed, or weren't sure, you then lookup the answer and more importantly, understand how to get it correct next time.

Take some time to do what you like. This test, and moreso studying for it, are emotionally taxing. There will be a day where you feel stupid, and that's okay.

Know your amino acids. Cold.

Did I mention experimental design?




7) How long did you study for the MCAT?
Three months, taking 11 credits. Working 24 hours a week, and Volunteering. Please do not do this to yourself. It sucked. I'm surprised I got the score I did with how much other stuff I had to worry about. I studied 6 hours a day during the week (yes, including school days) and did Khan videos during the weekend after my shift ended. The last month was crunch time as I was out of school. Probably 10 hours a day, five days a week.
 
1) 521 total 130 PC/129 CARS/130 Bio/132 Psy/Soc
2)Explicitly used Kaplan's materials and study guide for each section
3)I used only Kaplan and aamc material. Kaplan for content studying. Kaplan Q-bank and aamc Q packs for daily practice. Kaplan FLs 1-7. And AAMC FL right before exam. Also did the 120 Qs in the official guide to the MCAT.
4)Kaplan FLs 1-7 scoring ranged from 501-508. AAMC practice right before exam ~86%
5)Biology @ top 100 state school
6)I did every single little thing my Kaplan Course had to offer. I swear by them, they helped me so much. Which is why I now teach for them:) I strongly suggest taking a course. When you do, do every single reading, watch every single video, every practice Q they got. Suck everything you can out of that course.

My schedule was this. I followed their instructions to a Tee, except didn't just do "recommended" material, I did everything the course had to offer. around 5 hours a day. When the course was over, I still had 8 weeks till my exam. So I took a FL every weekend, and spent the rest of the week doing focused content on what I got wrong. And doing practice specific to that area. Thats it folks. I simply followed Kaplan, and did FLs after the course ended.

One thing I think REALLY heped.... do one practice passage from each of the 4 section EVERY DAY (except test days). I used the aamc Qpacks and official guide, and the Kaplan Q bank for these practice passages. Don't skip this ever. Do them. Do them Do them Do them. Oh one more thing. DO THEM. one P/C, one CARS, one BIO, and one Psy/soc every day.

5) 5 months for at least 5 hours a day...... yes folks... thats 5 x 30 x 5=750 hours... its actually more than that because FLs were 7 hours long.

Note: I'm not that bright... AT ALL. I do have a 3.99gpa, but thats only because I work myself to the bone!!!! Most people won't have to put in nearly 750 hours to do this well... probably around 300-350 from what I can see. BUT... if you're dumb like me... DON"T BE DISCOURAGED!!!! JUST WORK YOUR @$$ off. You can do it! WHO CARES IF THEY SAY 300-350 hours to prepare???!!! TRY TO DOUBLE THAT IF YOU KNOW YOUR NOT SMART!! I guarantee you can out work people smarter than you, and grab a disgusting score.

Good luck
 
1) Your individual scores and composite score
C/P: 132
CARS: 130
B/B: 131
P/S: 127
Composite: 520 (98th)
2) The study method used for each section

For C/P, B/B, and P/S, I simply did content review with EK followed by a ton of passages from pretty much any resource I could find, which were:
TPR Science Workbook (MCAT 2015 version)
All of the NextStep Strategy and Practice Books (These FL sections were tough, but I felt they were actually very good practice)
AAMC Official Guide
AAMC Study Packs
All of the FL's listed below
In addition, for P/S, I watched a lot (but not all of) the Khan Academy videos and took notes on them. I wish I had used all of the passages online through them, and finished watching all of the sociology videos. In addition, the glossary in the back of the TPR Psych book was useful in the final few days for me to make sure I had at least a passing idea of the definition of the key words mentioned throughout the book. As you can see, my P/S score was the lowest. However, as someone who has never taken a psychology or sociology class, I'm fine with what I ended up with (and it's not even a bad score...not about to complain haha)
3) What materials you used for each section(Kaplan, TPR, Examkrackers, AAMC, TBR, etc)
C/P:
Read through the EK Chemistry/Physics manuals, and did all the passages from the new MCAT 2015 TPR Science Workbook for the Ochem and GenChem questions. I did a few of the physics passages, but a lot of them seemed low yield to me, since they were extremely calculation based. Even though ochem is also relatively low yield, most of the passages were short. For any mechanisms that had given me trouble and I saw repeatedly popping up, I watched the corresponding Khan Academy videos and did a couple Khan passages. *ALSO* Even though the AAMC question packs are a different format to the current test, they're good for content review and reviewing concepts-especially those that you may need more practice on.
CARS:
Everyone has their own tricks here. The one that I found to work for me was to make sure I understand what each paragraph adds to the passage right after I read it. Instead of reading through and just saying "oh, ok", I forced myself to read critically and give a little summary at the end of each paragraph. The pace of reading slowed down, overall, but I would answer the questions much faster. This took me from a 9 on the old test to a 130, so I'd say it helped me out. I also read the EK advice on CARS, but I felt it didn't add much.
B/B:
Like for C/P, I read the corresponding EK books (Bio I and II), and then dove into the 2015 TPR Science Workbook bio passages. Honestly, I'm not sure how much these helped me. Sure, it's good to get a lot of bio practice for basic molecular biology and physiology memorization, but these passages don't really make you think critically like the real test will. In my opinion, I was most prepared for this section by my biochem course, as well as the several upper division molecular biology courses I had taken. The courses allowed me to be immersed in literature of the field, and taught me how to quickly analyze data and understand how research is approached and carried out. If you have time, take a molecular bio upper division course before taking this test. If not, read at least a few research articles that utilize common molecular bio techniques. This test truly has changed to favor molecular biology, and I believe that a lot more premeds in the future will be majoring in it in lieu of physiology....just wait. As with C/P, the AAMC question packs were good at reinforcing all of the content for various topics.
P/S: Honestly, there's probably better strategies out there besides mine. I started by reading the EK Psych book and quickly realized that it was severely lacking in information. I got a TPR Psych book, and really only used it for the FL's and the glossary at the big. Most of my psych studying came from Khan academy videos and my notes on them. Most of the concepts in psychology are relatively easy to memorize, but I personally think you need to be careful to not overlook sociology. I think that may have come back to haunt me a bit on my test. It's nice that the new Science Workbook by TPR has P/S passages in it, so get your hands on those if possible.
4) Which practice tests did you use? (Optional: include scores)
Kaplan 1: 503
TPR Demo: 503
TPR 1: 504
TPR 2: 507
EK1: 80%
EK2: 77%
EK3: 77%
EK4: 79%
NS1: 513-127/128/130/128
NS2: 509-128/127/128/126
NS3: 514-128/128/129/129
AAMC FL Take 1 (1st FL taken-1.5 months before test): 88%-93/87/86/81
AAMC FL Take 2 (3 days before test): 95%-95/94/95/95
OG: C/P-90% CARS-87% B/B-80% P/S-77%
5) What was your undergraduate major?
Molecular, Cell and Developmental Biology
6) Any other tips you may have for those of us who still have this test lurking over us?
You might notice a pattern, but make sure to do passages, passages, and more passages. Content review is important and memorization is certainly still needed for some questions you'll see on test day, but understanding the format and style of passages and questions is your most valuable asset.
Before you take this test, make sure part of the time you spend studying is fully dedicated to the MCAT!! When I took the old version, I was in school the whole time while studying, and barely pulled out a 30, which is still a decent score, yet I knew I could improve. This time around, I still did some studying during school, but also blocked off the first 6 weeks of my summer for strictly MCAT studying. Use the morning/afternoon to study, and unwind during the evening. This will prevent the burnout that happens to so many people. Lastly, give yourself at least one day off a week. Seriously. Just. Do it.
7) How long did you study for the MCAT?
A little over three months. I was also fresh out of biochem when I started studying, which was a huge help. I started off using the NextStep schedule, but felt that they spent too much time overall on content review.
(April 16th-July 17th)
If you want to see my study schedule for the last month or so leading into my test, just let me know! Hope this helps, and good luck!!!


Hi may I see your schedule if you still have it?
 
Hi there. Long-time lurker here; this is my first post. I’ve gotten so much great information off of this site, I figured I was long past due to give back.

[I should note here that I’m much older than your average pre-med — 39 — and I think this conveyed both advantages and disadvantages, which I’ll try to touch on at the appropriate time.]



1) Your individual scores and composite score

PS: 127
CARS: 132
BS: 131
Psych: 132

Total: 522 (99th percentile)



2) The study method used for each section

All sections
: I did every practice test I could get my hands on, especially the stuff released by the AAMC.

As well, a few friends and I did a course here in LA called Swartwood. I do NOT recommend this course. They are incredibly disorganized, paranoid, and outright bizarre. None of us — not one of us — was glad we took this course. (I do think that their method makes sense for CARS, though. I’ll explain below.)


PS: This has always been my Achilles heel (due to various reasons including the fact that I have some weird mental problem with basic arithmetic… I’m quite capable of adding 2 + 3 and getting 6). So, I pushed myself harder in this section than in any other. Luckily, it was more conceptual and less math-y than I expected, though you do have to have those equations memorized, because you never know which one will come up!

I made flash cards for the equations. The physics stack was huge, but I just made myself drill it (alone as well as with friends). The gen chem stack was not so big. I also glanced over solubility rules and such just to make sure I remembered them, but didn’t memorize them flat-out.

Other than that: practice practice practice. I went back over anything that felt rusty from my classes, and really tried to avoid plug-and-chug. What do I mean by this? I mean that, tempting as it was at times, I never allowed myself to just throw in an equation and get a result without knowing why that was the result. This is very very important for the MCAT style of reasoning.

Also (and this applies to both PS and BS), I memorized the **** out of the amino acids. I used a couple of ipad apps — both entitled “Amino Acids," helpfully — and tried to be able to recognize them whether they were drawn in bond-line, 3D, “old-school” Lewis-ish notation, etc. As well, there are some unique features to certain amino acids, such as glycine being achiral and ambivalent, or proline causing kinks in peptide chains. Know those. And know the single-letter codes! I messed up on that.


CARS: This is one of the places where I think my age came in handy. I’ve been a lifelong reader, so I had a sort of store of “crystallized knowledge” upon which to draw. Other reading-oriented friends (though younger) had a similarly good time with CARS. I’ll admit to not really studying for this section, other than by doing the practice tests.

A passing familiarity with a wide variety of subjects is a boon, but it’s admittedly difficult to learn “a little about a lot” without having time. With that said, however, there is a method (gleaned from Swartwood) that can help those among you who don’t have time for a lot of casual reading (and as pre-meds, that’s probably most of you): Keep it stupid!

I took notes that were really, really dumb. Like, first paragraph: “guy likes Rembrant.” Second paragraph: “Other guy thinks first guy sux.” Literally, I wrote stuff like that. Just try to boil each paragraph down to its essential characteristic, in the most childish, unremarkable terms possible.
Also, If you find a metaphor used over and over again in a passage, hone in on that. It’s probably important.

For some context, I have a super math-oriented friend who hates reading, and using the stupidity method, he was able to boost his scores considerably over time. He just had to stop trying so hard.


BS: See above regarding the amino acids. Applies to both sections. Other than that, I refamiliarized myself with cellular stuff, DNA/RNA, basic physiology. The kidney can be kind of complex, make sure you understand what's going on there. Also, know what actions take place in which part of the digestive system. I was a psych major, so I already had a passing familiarity with neuro stuff, but they definitely seemed to like this topic as well.

But definitely, definitely study a lot of biochem. They went hard on it. Know the essentials: which steps of glycolysis are irreversible, for example. Anything unusual or that ”sticks out,” try to memorize.

We made flash cards for the hormones, cell types, and steps in spermatogenesis and oogenesis.


Psych: I was a psych major (having completed my major only two years ago), so I was kind of ”psyched” on this section. :naughty:
I did the practice tests and made sure to memorize key terms (as laid out in the ExamKrackers books).



3) What materials you used for each section(Kaplan, TPR, Examkrackers, AAMC, TBR, etc)

As mentioned above, we did every (relevant) AAMC test. They put out bundles collected from older tests; we did those. The 120 questions; we did those. The “actual” practice tests; definitely did those. Some parts we did twice, just in case.

Otherwise: The ExamKrackers books were great, though I feel they were a bit lax in the biochem department. Also, they seemed to be guessing a bit in psych, but I think that was the case with everybody. As time goes on and more tests are released, prep companies will get a better feel for what that section looks like.

We also had some tests from Swartwood, which were… okay, I guess. They were riddled with typos and often had wrong answers. Also, they couldn’t get their act together in time to give us computer-based tests, so we did them on paper. And out of order. With long-ass breaks. And sometimes not all the same sections on the same day. Kind of wack, but it got us thinking about the material, so it was still sorta useful in that sense.

Khan academy has some great info and good practice tests, but they’re way too broad in scope. They need to narrow it down a bit (which I’m sure will happen with time). But I found their stuff useful for finding out more about specific topics about which I was confused or felt under-prepared. In other words, come to Khan somewhat primed and be targeted, otherwise you could get lost in the mountain of information.

Princeton has one free test, and we did that. It was horrible. Horrible! Demoralizing. Awful. And 90 times harder than the real MCAT. Like, it was crazy.

One thing that was useful was that we combined material to simulate test conditions. So, we’d take 52-ish questions from an old AAMC Physics section, same from old Verbal and Bio sections, and combine them with ExamKrackers psych to make a “whole” test. It was a way to not only study material, but to get the timing down. More about this later.



4) Which practice tests did you use? (Optional: include scores)

Oops. See above.

I don’t remember my scores. I feel like they were decent, but lower than on test day overall, with the possible exception of the 120 questions. Those were hard.



5) What was your undergraduate major?

Psych, with a few extra neuro courses thrown in for good measure.



6) Any other tips you may have for those of us who still have this test lurking over us?

Yes:

I think that some of the prep companies may be overestimating the complexity of the questions on the MCAT. They’ve all been racing to catch up to the new format and stylistic changes (some small, some big), and it’s a bit chaotic out there. But one thing I noticed was that most of their material was harder than the actual MCAT. So, don’t be discouraged if your practice scores are less than you want/expect. Stay strong and keep working! You CAN do this, it’s doable. And you may just be surprised by the outcome.

My exam had a heavy biochem emphasis in both the PS and BS sections. Study that ****. If you haven’t taken a course in it, you;ll be at a disadvantage, but can probably still learn the essentials.

Definitely memorize the amino acids!

I definitely think that some random “experimental” problems turned up. If it’s not covered in the practice materials (or the AAMC syllabus, which I barely touched), try quickly and move on. For example, if something were to show up about (hypothetically) about diffraction gratings where they don’t give you any equations (hypothetically), I wouldn’t sweat it.

Also, remember: On test day, everyone is just as nervous as you are! So even though the test is not “curved” as such, your percentile may end up being higher than you think.

I found that the psych threw a lot of people off. I think as more universities acclimate to this new MCAT, they’ll put more premed-oriented psych classes in place. In the meantime, focus on the terms, and the rest is critical thinking. And a lot of common sense.

(Actually, that may be a good thing to focus on: Figure out what parts of psych diverge from common sense, and make sure you remember those.)

PS: I never took a sociology course, nor did I take social psych. Ain’t necessary.

Watch out for fatigue during CARS. Have a small snack beforehand, even if you’re not hungry. Pee, even if you don’t think you have to.

And speaking of that stuff, have a good breakfast, lots of protein. I had eggs and a piece of fruit. Little bit of caffeine. (I had too much caffeine during lunch and was over-jacked on the bio section, so be careful)…

This is important: If your test is at 8am, get up early way in advance! I wanted to make sure I had zero “fuzz” on the brain at 8am, so I practiced getting up earlier and earlier as time went on. By the week before the test, I was waking up at 5:30 every day. it sucked but I think it helped, because I was well into mid-morning by the time 8am came around, and had had time to eat properly, shower, and feel generally as fresh as possible.

GET THERE EARLY. Rushing will throw you way off, emotion- and stress-wise!

Also, and this is very important (IMHO)… review your tests with your friends/study buddies! We went over everything that any of us (3 or 4 of us) got wrong, and it was immensely helpful. Guaranteed, someone in your group will see something differently, have some insight, or otherwise be able to point out something you missed. And it will stick.

One last thing: The highlighting was slightly different on the real test. You have to click an extra time to highlight and de-highlight things, for some reason. Not a game-changer, but slightly annoying.



7) How long did you study for the MCAT?

Prep course Feb-May (took it May 22nd), ramped up big time towards the last week. Then barely studied on the second-to-last day before the test, other than to go over some equations. Day before, went to the movies, relaxed.



Congrats, this is a big help.
 
1) Your individual scores and composite score

C/P: 129 CARS: 127 B/B: 132 P/S: 132
Composite: 520
2) The study method used for each section

This was my weekly schedule:

Saturday: take full-length practice exam starting at 8 am (simulate test day conditions, no phone usage, take the full breaks)

Sunday: fill out WIMI (why I missed it) sheets for the chem/phys section of Saturday's exam. Try to understand each question you missed and review the ones you got right. Take organized notes. Spend the rest of the day reviewing chem/phys. "Why I missed it sheets" just means a word document or handwritten notes. Practice problems are very important here.

Monday: make WIMI sheets for CARS. Re-read passages and practice outlining. Try to understand what the author's message is. Try to understand what each question is asking. Do 5-10 practice cars passages. Time yourself and try to finish each one in 8-10 minutes or less! Spend the rest of the day with weak chem/phys/bio/biochem/psych/soc areas.

Tuesday: Make bio/biochem WIMI sheets. Be very detailed in notes! Spend the rest of the day studying bio/biochem.

Wednesday: Make psych/soc WIMI sheets. Be very detailed in notes! Spend the rest of the day studying psych-soc.

Thursday/Friday: HIT THOSE WEAK AREAS! Make flash cards for chem/phys equations and psych/soc terms. If you nail vocab then psych is easy. Do practice problems!!!!

Friday evening: relax after dinner and go to bed early.

IN SUMMARY
Take a practice test every 6-7 days.
Review each section of the practice exams thoroughly with WIMI sheets.
Videos/books/notes to review weaker areas.
Do 2-3 cars passages EVERY DAY.
Try to get 7+ hours of sleep.
Use the AAMC list of psych/soc terms to make sure you hit all the content.
Take AT LEAST 10 practice exams from multiple companies (I used kaplan exclusively but I wish I would have taken exams from other companies as well).

3) What materials you used for each section(Kaplan, TPR, Examkrackers, AAMC, TBR, etc)

Kaplan, exclusively.

4) Which practice tests did you use? (Optional: include scores)

Kaplan Diagnostic: 495
Kaplan Full-length 1: 496
Kaplan Full-length 2: 503
Kaplan Full-length 3: 505
Kaplan Full-length 4: 506
Kaplan Full-length 5: 506
Kaplan Full-length 6: 508
Kaplan Full-length 7: 507
Kaplan Full-length 8: 505
Kaplan Full-length 9: 509 *2 weeks before MCAT
AAMC Scored Exam: 516 *5 days before MCAT
MCAT: 520

5) What was your undergraduate major?

Biomedical Sciences with a Chemistry Minor

6) Any other tips you may have for those of us who still have this test lurking over us?

DO NOT STRESS. I had 2 hours of sleep the night before. Not good at all. Listen to relaxing music and talk to friends the day before.
Ideally, you should try to take 15 practice exams and study at least 600+ hours for at least 6 months.
In hindsight, I did too much content review and not enough practice problems. My CARS timing was poor which led to the 127. Had I done more practice problems, my timing would have improved.

7) How long did you study for the MCAT?

164 total days from the end of January to July 9th.
517.4 total hours.
 
Last edited by a moderator:
  • Like
Reactions: 9 users
1) Your individual scores and composite score
Took it twice

5/20/2016
509/129/125/128/127
7/9/2016 514/130/127/129/128

2) The study method used for each section
C/P: I was very strong in O-chem and Biochemistry so I would just read Lehninger's principles of Biochemistry 6th edition to keep reviewing the topics. GC and Physics were the subjects I needed the most work in so I got better at them by doing TONS of practice passages. For C/P that is really all you need to do to get better. Do lots of practice passages that specifically target your weak area.

CARS: I read the EK booklet and would read the passages for the main idea. I would have an internal conversation in my head about the author. I formed images of him/her and would curse him/her out in my head. "This mother****er is a dumb**** for making this argument. X, Y and Z all show he is a ****ing ******. **** his ideas. You get the picture, I would do everything in my power to keep myself interested in the passage. I found that I got better at the reasoning beyond the text questions once I started making my own analogies in my head to what they were saying. Like, Picasso was a genius for his style - I might think "Picasso, huh didn't I see some of his pictures and didn't they make me think that X, Y , and Z". Point being is that by making CARS relevant to me I was better able to stay interested in the long boring Lit/Philosophy passages I got on Test day.
Practice on computers should be done with 1280x800 resolution to mimic test day conditions. You want enlarged font so the passages don't seem like they are 4x as long.

Here is how I reviewed CARS answers for my retake
This is really where you start improving:
  1. Why did you pick the wrong answer
  2. Where in the passage is the correct answer, does it tie into the main idea.
  3. Why is the answer you picked wrong.
  4. Why is the correct answer the best answer. Where in the passage can you find evidence that this is the best answer.
  5. What was the author's main idea, can it be related to this question.
Also what type of a question was it (reasoning beyond the text, strengthen/weaken, roman numeral etc.) make a list of these. I always missed Those types until I started analyzing them on why I got them wrong.

B/B: I have a very weak Biology Background and my baseline for this section was a 124/125 and only because I'm good at Biochem. I studied more for this section than all the rest combined because of the sheer amount of information. My weak areas were the dense SB style Molecular Biology passages so I practiced getting better at the million acronym passages by using my scratch paper. If you see a passage with a million enzymes you WILL NOT remember them all unless you are a ****ing god. I would write down what they did and draw arrows depending on if they activated/deactivated something or if their levels increased in the presence of something. Expect these passages on test day and I highly suggest you practice like this. I really thought I was going to do better in this section but oh well. For someone who has only taken Gen Bio 1 and 2 and Biochemistry 1 I really hadn't covered a lot of the material. I learned all the physiology by doing lots of passages since this was a weak area for me as well. I highly suggest that you learn this section by doing lots of passages. B/B is much more Experimental based and is extremely reasoning based. By doing passages during the content stage you solidify these concepts and improve your reasoning skill.

P/S: I never took Psychology or Sociology and only survived this section by applying my CARS skills to this section and by doing lots of practice. Some people claim this section is just route memorization of terms but I'm going to disagree. It is very experimental based so I would really only practice this section by doing lots of FLs. EK P/S was pretty good practice but nothing compares to AAMC. If I could redo it, this section would be what I would change the most. I would just pick a prep book, go through it in detail and supplement it with the KA 100 page guide.


3) What materials you used for each section(Kaplan, TPR, Examkrackers, AAMC, TBR, etc)
C/P: EK, TPR GC and Physics, Lehninger's Principles of Biochemistry and all AAMC
CARS - Old AAMC VR, EK CARS book, TPR Online practice problems/Hyperlearning book.
B/B - all AAMC, EK Bio 1 and 2 (excellent), Lehninger Principles of Biochemistry 6th, TPR Biology
P/S - all AAMC, KA 100 page guide, EK Psychology book and some TPR Psychology book, as well as a Psychology Textbook I bought cheap for referencing all the terms I didn't know.

4) Which practice tests did you use? (Optional: include scores)
Test: Overall C/P CARS B/B P/S Date
NS Diagnostic 502/125/126/125/126 3/8/2016
TPR Complete 3 497/125/123/125/124 3/12/2016
TPR Demo 501/123/126/125/127 3/19/2016
AAMC 1/2 Test 75%/87%/67%/70%/77% 5/10/2016
AAMC Scored 507/128/127/128/124 5/11/2016
Kaplan FL 3 501/125/125/127/124 5/13/2016
AAMC Sample Test 77%/83%/74%/78%/75% 5/18/2016
AAMC 5/20/2016 509/129/125/128/127 5/20/2016
TPR Complete 1 510/127/125/129/129 6/1/2016
NS FL I 510/128/126/128/128 6/9-10/2016
Kaplan FL 1 505/127/126/127/125 6/18/2016
AAMC 10 37/12/10/15 6/22/2016
AAMC 3 VR = 11 6/26/2016
AAMC 4 VR= 12 6/28/2016
EK FL 1 70%/66%/72%/66%/76% 6/25/2016
EK FL 2 74%/64%/74%/75%/83% 6/30-7/3
OG R2 91%/93%/93%/90%/87% 7/1/2016
AAMC Sample Test R2 92%/97%/94%/88%/88% 7/2/2016
AAMC Practice Scored 2 523/132/131/131/129 7/4/2016
EK FL 3 75%/77%/90%/x 7/3-5/2016
EK FL 4 85%/x/92%/80% 7/6/2016

Mid to high 80s on all question packs other than CARS 1 before my first mcat. SB C/P 90%, SB B/B 78, P/S = 61 all before first MCAT. Redid B/B and C/P and got 93% on both a few days before retake and got a 88% on CARS 2 which I did for the first time before my retake.

5) What was your undergraduate major?
Biochemistry

6) Any other tips you may have for those of us who still have this test lurking over us?
AAMC material over everything. Ignore your test prep scores, especially for CARS. How you feel going into the exam will affect your performance. If you feel like you are going to bomb it or get flustered by a crazy hard C/P section it will affect the rest of your performance. On my first MCAT I got an insanely difficult C/P and feel like I missed 15-20+. I thought for sure I was bombing that section and went into CARS not feeling good and anxious as ****. I let C/P ruin CARS and the rest of that exam. Do not do that. Your mental state in CARS will determine your score. Even during my second test I had a moment of panicky anxiety because it was HARD. Like insanely hard, way worse than any practice material. I'm convinced the momentary shakeup is why I didn't hit 130+, I probably messed up that passage. Also recommend that you learn Biochemistry very very well. Obviously the basics like amino acids and protein structure. But the more of Lehninger's principles of Biochemistry you know the less scary the exam will feel. I know for a fact that almost every topic in B/B and C/P could be found somewhere in that passage. Know your Biochemistry Lab techniques/research design because those tough skill 3 and skill 4 questions will becomes much easier to answer.

How you feel coming out of the exam does not reflect how you will actually do. I thought on my retake I was getting a 132 in C/P and 131-132 in B/B and thought I bombed P/S ~126. I was wrong on all accounts. Also first MCAT felt like I was getting a 125-126 in C/P and got a 129. You can never tell how you are going to do and can only hope for the best. Also if you aren't satisfied with your score and it is under 515 and feel like you can do better, don't let anyone tell you to settle, retake the sucker because you will feel much better with a score increase.


Here is my general strategy I recommend to people with a month-2 months left who finished content review:

Day 1: Take a FL
Day 2: Review the FL and do a practice CARS section. Light content review
Day 3: Practice problems in your weak areas from the FL and some content review in those area.

Repeat cycle

Day 7: Light content review/cheat day/ whatever you want.

I really believe that this schedule can work for anyone and that the more practice you do the better off you will be. I spent too long on content review because I hadn't taken all the UG classes. If I was able to do this schedule full time I feel like I could have done better and really believe that it is flexible for anyone regardless how smart you are. If you want to increase your score you need to practice and improve your weak areas. Sure you could be a god at O-chem and get every passage right but if you suck at physics and miss every other question you won't get a great score that you have the potential to get. Yeah I know, we all hate studying our weak areas. Do it. I'll say it again Study your ****ing weak areas.

So what practice resources should we be using?
Honestly, what you use isn't as important as people think. I thought TPR was fine for GC and Physics content practice. Yeah there isn't experimental passages but I was trying to iron out my weak areas.
Use all AAMC material. This means that I don't think you should do the CARS question packs. You should really being doing the old VR sections. You will have 9 practice sections vs 4 where you actually will get a score to accurately show where you are. Compare your score with the percentiles between the old and new exam for VR. I also suggest doing the Bio and P/S sections, especially Bio because of all the O-chem passages in there.

So which FLs do reccomend for the 3 day cycle?

EK1-4, EK4 is the best followed by EK3. Afterwords get the NS tests.

But I'm broke how can afford all this stuff?
Okay so NS 1 and the diagnostic are free. You can probably find someone to sell you access for cheap.

I'm a retaker and have exhausted all CARS resources? Help?

Okay, there are 6 free TPR tests and 3 Kaplan tests. CARS hasn't changed that much from the old VR section which means you can rely on TPR more here than for the other sections. Get the isbns for both and register them and you have 9 practice tests. This would be helpful for Day 2 of my cycle where you have to do practice CARS.

Also take some time in the evening of each day to do something fun. You don't have to study every second. Even when I studied for 6-8 A.M, worked from 8:30-5, studied from 6-10 (month of June) I still managed to binge watch T.V from 10-11:30/12 or go out to dinner with someone etc. If you don't take a break your mental state will crumble and you will lose time on your studying (me in the month of April). Also Do not underestimate this test, especially CARS. I don't care how good of a reader you are, that you have an english degree, or that you have taken tons of liberal arts classes (I sure have). MCAT verbal is a whole different beast and you need to be daily practicing if you want to break into the upper echelon.

Also I did think I was going to score higher on the retake but once you get to the 129+ area it comes down to luck with your guesses and I guess mine weren't so hot.

7) How long did you study for the MCAT?
I really studied for the MCAT from May (fulltime) until July (mainly part-time in June 4-8 hours a day. I really wished I didn't take the month of April off (personal stuff) because then I wouldn't have had to study while doing 9 hours of research a day.


Final Notes:

I am sure I will be editing this for more useful tidbits but this is all for now. Good luck all, if I can break 80% so can you!
 
Last edited:
  • Like
Reactions: 3 users
Hi, folks. I already took the MCAT, but I'm curious, when you guys say you're "content reviewing" for X amount of months, are you just reading or are you reading and doing the accompanying problems afterward and any other freestanding problems and passage based after that from the book or any other source? and when you say you're really doing practice after that, are you talking specifically full lengths afterward? Sorry if it sounds stupid.
 
1) Your individual scores and composite score

C/P: 129 CARS: 127 B/B: 132 P/S: 132
Composite: 520
2) The study method used for each section

This was my weekly schedule:

Saturday: take full-length practice exam starting at 8 am (simulate test day conditions, no phone usage, take the full breaks)

Sunday: fill out WIMI (why I missed it) sheets for the chem/phys section of Saturday's exam. Try to understand each question you missed and review the ones you got right. Take organized notes. Spend the rest of the day reviewing chem/phys. "Why I missed it sheets" just means a word document or handwritten notes. Practice problems are very important here.

Monday: make WIMI sheets for CARS. Re-read passages and practice outlining. Try to understand what the author's message is. Try to understand what each question is asking. Do 5-10 practice cars passages. Time yourself and try to finish each one in 8-10 minutes or less! Spend the rest of the day with weak chem/phys/bio/biochem/psych/soc areas.

Tuesday: Make bio/biochem WIMI sheets. Be very detailed in notes! Spend the rest of the day studying bio/biochem.

Wednesday: Make psych/soc WIMI sheets. Be very detailed in notes! Spend the rest of the day studying psych-soc.

Thursday/Friday: HIT THOSE WEAK AREAS! Make flash cards for chem/phys equations and psych/soc terms. If you nail vocab then psych is easy. Do practice problems!!!!

Friday evening: relax after dinner and go to bed early.

IN SUMMARY
Take a practice test every 6-7 days.
Review each section of the practice exams thoroughly with WIMI sheets.
Videos/books/notes to review weaker areas.
Do 2-3 cars passages EVERY DAY.
Try to get 7+ hours of sleep.
Use the AAMC list of psych/soc terms to make sure you hit all the content.
Take AT LEAST 10 practice exams from multiple companies (I used kaplan exclusively but I wish I would have taken exams from other companies as well).

3) What materials you used for each section(Kaplan, TPR, Examkrackers, AAMC, TBR, etc)

Kaplan, exclusively.

4) Which practice tests did you use? (Optional: include scores)

Kaplan Diagnostic: 495
Kaplan Full-length 1: 496
Kaplan Full-length 2: 503
Kaplan Full-length 3: 505
Kaplan Full-length 4: 506
Kaplan Full-length 5: 506
Kaplan Full-length 6: 508
Kaplan Full-length 7: 507
Kaplan Full-length 8: 505
Kaplan Full-length 9: 509 *2 weeks before MCAT
AAMC Scored Exam: 516 *5 days before MCAT
MCAT: 520

5) What was your undergraduate major?

Biomedical Sciences with a Chemistry Minor

6) Any other tips you may have for those of us who still have this test lurking over us?

DO NOT STRESS. I had 2 hours of sleep the night before. Not good at all. Listen to relaxing music and talk to friends the day before.
Ideally, you should try to take 15 practice exams and study at least 600+ hours for at least 6 months.
In hindsight, I did too much content review and not enough practice problems. My CARS timing was poor which led to the 127. Had I done more practice problems, my timing would have improved.

7) How long did you study for the MCAT?

164 total days from the end of January to July 9th.
517.4 total hours.

What was your content review like??
 
1) Your individual scores and composite score
Took it twice

5/20/2016
509/129/125/128/127
7/9/2016 514/130/127/129/128

2) The study method used for each section
C/P: I was very strong in O-chem and Biochemistry so I would just read Lehninger's principles of Biochemistry 6th edition to keep reviewing the topics. GC and Physics were the subjects I needed the most work in so I got better at them by doing TONS of practice passages. For C/P that is really all you need to do to get better. Do lots of practice passages that specifically target your weak area.

CARS: I read the EK booklet and would read the passages for the main idea. I would have an internal conversation in my head about the author. I formed images of him/her and would curse him/her out in my head. "This mother****er is a dumb**** for making this argument. X, Y and Z all show he is a ****ing ******. **** his ideas. You get the picture, I would do everything in my power to keep myself interested in the passage. I found that I got better at the reasoning beyond the text questions once I started making my own analogies in my head to what they were saying. Like, Picasso was a genius for his style - I might think "Picasso, huh didn't I see some of his pictures and didn't they make me think that X, Y , and Z". Point being is that by making CARS relevant to me I was better able to stay interested in the long boring Lit/Philosophy passages I got on Test day.

B/B: I have a very weak Biology Background and my baseline for this section was a 124/125 and only because I'm good at Biochem. I studied more for this section than all the rest combined because of the sheer amount of information. My weak areas were the dense SB style Molecular Biology passages so I practiced getting better at the million acronym passages by using my scratch paper. If you see a passage with a million enzymes you WILL NOT remember them all unless you are a ****ing god. I would write down what they did and draw arrows depending on if they activated/deactivated something or if their levels increased in the presence of something. Expect these passages on test day and I highly suggest you practice like this. I really thought I was going to do better in this section but oh well. For someone who has only taken Gen Bio 1 and 2 and Biochemistry 1 I really hadn't covered a lot of the material. I learned all the physiology by doing lots of passages since this was a weak area for me as well. I highly suggest that you learn this section by doing lots of passages. B/B is much more Experimental based and is extremely reasoning based. By doing passages during the content stage you solidify these concepts and improve your reasoning skill.

P/S: I never took Psychology or Sociology and only survived this section by applying my CARS skills to this section and by doing lots of practice. Some people claim this section is just route memorization of terms but I'm going to disagree. It is very experimental based so I would really only practice this section by doing lots of FLs. EK P/S was pretty good practice but nothing compares to AAMC. If I could redo it, this section would be what I would change the most. I would just pick a prep book, go through it in detail and supplement it with the KA 100 page guide.


3) What materials you used for each section(Kaplan, TPR, Examkrackers, AAMC, TBR, etc)
C/P: EK, TPR GC and Physics, Lehninger's Principles of Biochemistry and all AAMC
CARS - Old AAMC VR, EK CARS book, TPR Online practice problems/Hyperlearning book.
B/B - all AAMC, EK Bio 1 and 2 (excellent), Lehninger Principles of Biochemistry 6th, TPR Biology
P/S - all AAMC, KA 100 page guide, EK Psychology book and some TPR Psychology book, as well as a Psychology Textbook I bought cheap for referencing all the terms I didn't know.

4) Which practice tests did you use? (Optional: include scores)
Test: Overall C/P CARS B/B P/S Date
NS Diagnostic 502/125/126/125/126 3/8/2016
TPR Complete 3 497/125/123/125/124 3/12/2016
TPR Demo 501/123/126/125/127 3/19/2016
AAMC 1/2 Test 75%/87%/67%/70%/77% 5/10/2016
AAMC Scored 507/128/127/128/124 5/11/2016
Kaplan FL 3 501/125/125/127/124 5/13/2016
AAMC Sample Test 77%/83%/74%/78%/75% 5/18/2016
AAMC 5/20/2016 509/129/125/128/127 5/20/2016
TPR Complete 1 510/127/125/129/129 6/1/2016
NS FL I 510/128/126/128/128 6/9-10/2016
Kaplan FL 1 505/127/126/127/125 6/18/2016
AAMC 10 37/12/10/15 6/22/2016
AAMC 3 VR = 11 6/26/2016
AAMC 4 VR= 12 6/28/2016
EK FL 1 70%/66%/72%/66%/76% 6/25/2016
EK FL 2 74%/64%/74%/75%/83% 6/30-7/3
OG R2 91%/93%/93%/90%/87% 7/1/2016
AAMC Sample Test R2 92%/97%/94%/88%/88% 7/2/2016
AAMC Practice Scored 2 523/132/131/131/129 7/4/2016
EK FL 3 75%/77%/90%/x 7/3-5/2016
EK FL 4 85%/x/92%/80% 7/6/2016

Mid to high 80s on all question packs other than CARS 1 before my first mcat. SB C/P 90%, SB B/B 78, P/S = 61 all before first MCAT. Redid B/B and C/P and got 93% on both a few days before retake and got a 88% on CARS 2 which I did for the first time before my retake.

5) What was your undergraduate major?
Biochemistry

6) Any other tips you may have for those of us who still have this test lurking over us?
AAMC material over everything. Ignore your test prep scores, especially for CARS. How you feel going into the exam will affect your performance. If you feel like you are going to bomb it or get flustered by a crazy hard C/P section it will affect the rest of your performance. On my first MCAT I got an insanely difficult C/P and feel like I missed 15-20+. I thought for sure I was bombing that section and went into CARS not feeling good and anxious as ****. I let C/P ruin CARS and the rest of that exam. Do not do that. Your mental state in CARS will determine your score. Even during my second test I had a moment of panicky anxiety because it was HARD. Like insanely hard, way worse than any practice material. I'm convinced the momentary shakeup is why I didn't hit 130+, I probably messed up that passage. Also recommend that you learn Biochemistry very very well. Obviously the basics like amino acids and protein structure. But the more of Lehninger's principles of Biochemistry you know the less scary the exam will feel. I know for a fact that almost every topic in B/B and C/P could be found somewhere in that passage. Know your Biochemistry Lab techniques/research design because those tough skill 3 and skill 4 questions will becomes much easier to answer.

How you feel coming out of the exam does not reflect how you will actually do. I thought on my retake I was getting a 132 in C/P and 131-132 in B/B and thought I bombed P/S ~126. I was wrong on all accounts. Also first MCAT felt like I was getting a 125-126 in C/P and got a 129. You can never tell how you are going to do and can only hope for the best. Also if you aren't satisfied with your score and it is under 515 and feel like you can do better, don't let anyone tell you to settle, retake the sucker because you will feel much better with a score increase.


Here is my general strategy I recommend to people with a month-2 months left who finished content review:

Day 1: Take a FL
Day 2: Review the FL and do a practice CARS section. Light content review
Day 3: Practice problems in your weak areas from the FL and some content review in those area.

Repeat cycle

Day 7: Light content review/cheat day/ whatever you want.

I really believe that this schedule can work for anyone and that the more practice you do the better off you will be. I spent too long on content review because I hadn't taken all the UG classes. If I was able to do this schedule full time I feel like I could have done better and really believe that it is flexible for anyone regardless how smart you are. If you want to increase your score you need to practice and improve your weak areas. Sure you could be a god at O-chem and get every passage right but if you suck at physics and miss every other question you won't get a great score that you have the potential to get. Yeah I know, we all hate studying our weak areas. Do it. I'll say it again Study your ****ing weak areas.

So what practice resources should we be using?
Honestly, what you use isn't as important as people think. I thought TPR was fine for GC and Physics content practice. Yeah there isn't experimental passages but I was trying to iron out my weak areas.
Use all AAMC material. This means that I don't think you should do the CARS question packs. You should really being doing the old VR sections. You will have 9 practice sections vs 4 where you actually will get a score to accurately show where you are. Compare your score with the percentiles between the old and new exam for VR. I also suggest doing the Bio and P/S sections, especially Bio because of all the O-chem passages in there.

So which FLs do reccomend for the 3 day cycle?

EK1-4, EK4 is the best followed by EK3. Afterwords get the NS tests.

But I'm broke how can afford all this stuff?
Okay so NS 1 and the diagnostic are free. You can probably find someone to sell you access for cheap.

I'm a retaker and have exhausted all CARS resources? Help?

Okay, there are 6 free TPR tests and 3 Kaplan tests. CARS hasn't changed that much from the old VR section which means you can rely on TPR more here than for the other sections. Get the isbns for both and register them and you have 9 practice tests. This would be helpful for Day 2 of my cycle where you have to do practice CARS.

Also take some time in the evening of each day to do something fun. You don't have to study every second. Even when I studied for 6-8 A.M, worked from 8:30-5, studied from 6-10 (month of June) I still managed to binge watch T.V from 10-11:30/12 or go out to dinner with someone etc. If you don't take a break your mental state will crumble and you will lose time on your studying (me in the month of April). Also Do not underestimate this test, especially CARS. I don't care how good of a reader you are, that you have an english degree, or that you have taken tons of liberal arts classes (I sure have). MCAT verbal is a whole different beast and you need to be daily practicing if you want to break into the upper echelon.

Also I did think I was going to score higher on the retake but once you get to the 129+ area it comes down to luck with your guesses and I guess mine weren't so hot.

7) How long did you study for the MCAT?
I really studied for the MCAT from May (fulltime) until July (mainly part-time in June 4-8 hours a day. I really wished I didn't take the month of April off (personal stuff) because then I wouldn't have had to study while doing 9 hours of research a day.


Final Notes:

I am sure I will be editing this for more useful tidbits but this is all for now. Good luck all, if I can break 80% so can you!
Hey congrats!! Do you think EK exams are harder than the real exam? I just took EK1 (62%, I felt C/P was a complete crap) and I did worse than EK3 &4( 88% C/P on EK4 and overall 70%). I feel like the way they state the questions are completely different and more complex than AAMC. I will be taking the exam on the 25th. I will be starting AAMC material tomorrow.
 
Hey congrats!! Do you think EK exams are harder than the real exam? I just took EK1 (62%, I felt C/P was a complete crap) and I did worse than EK3 &4( 88% C/P on EK4 and overall 70%). I feel like the way they state the questions are completely different and more complex than AAMC. I will be taking the exam on the 25th. I will be starting AAMC material tomorrow.
Thank you! I thought the real exam was harder than all the practice tests too be honest but I'm sure as you can see by my scores, I did better on the real exam. EK C/P for EK1 and EK2 were very difficult and I did worse on them and found them harder than my July 9th's C/P but easier than the May 20th one. Its hit or miss, just realize that the scaling/curve will reflect the difficulty of the exam you got. I got a very similar score for the most part on the real MCAT as I got on the EK4 test so it was a fairly accurate predictor for C/P, B/B and P/S using my conversion I made. I agree with you about EK tests, they were excellent practice.
 
  • Like
Reactions: 1 user
Thank you! I thought the real exam was harder than all the practice tests too be honest but I'm sure as you can see by my scores, I did better on the real exam. EK C/P for EK1 and EK2 were very difficult and I did worse on them and found them harder than my July 9th's C/P but easier than the May 20th one. Its hit or miss, just realize that the scaling/curve will reflect the difficulty of the exam you got. I got a very similar score for the most part on the real MCAT as I got on the EK4 test so it was a fairly accurate predictor for C/P, B/B and P/S using my conversion I made. I agree with you about EK tests, they were excellent practice.
So its all depends on luck!!!! Ugh!!! What about B/B? My B/B score varies across all my exams lol. And CARS always depends on how motivated i am as you mentioned. Does it help to do the AAMC material twice? Plus you mentioned that no test company gets close to the real exam and you did much better on the real exam than on EK
 
Hi, folks. I already took the MCAT, but I'm curious, when you guys say you're "content reviewing" for X amount of months, are you just reading or are you reading and doing the accompanying problems afterward and any other freestanding problems and passage based after that from the book or any other source? and when you say you're really doing practice after that, are you talking specifically full lengths afterward? Sorry if it sounds stupid.
I'm sure everyone is different, but when I refer to 'content review,' it was just reading. I didn't have time to do the accompanying problems!
For me, 'practice' was full lengths, as well as Q packs, section banks, etc.
 
  • Like
Reactions: 1 users
I'm sure everyone is different, but when I refer to 'content review,' it was just reading. I didn't have time to do the accompanying problems!
For me, 'practice' was full lengths, as well as Q packs, section banks, etc.

Thanks for the input. I'm consider retaking. Just like you, when I was reviewing I only "read" the chapters....and did the little chapter problems that came with Kaplan. But it seems that the majority of the successful folks are doing more problems as they're reading along? IDK I'm kinda confused.
 
So its all depends on luck!!!! Ugh!!! What about B/B? My B/B score varies across all my exams lol. And CARS always depends on how motivated i am as you mentioned. Does it help to do the AAMC material twice? Plus you mentioned that no test company gets close to the real exam and you did much better on the real exam than on EK
It's not necessarily luck -- I thought I got extremely unlucky on the C/P with my difficult section, and ended up scoring a 131. Marked 20+ questions. And, for what it's worth, I thought the B/B on my test was easier than both C/P and P/S, but it was my lowest subscore. My guess is that the curve was much harsher, and that you just have to trust that the curve will do it's magic.. I didn't do the AAMC material twice, so I can't speak to that, but my actual score very closely aligned with the AAMC scored. I would only do the AAMC FLs once.
 
Thanks for the input. I'm consider retaking. Just like you, when I was reviewing I only "read" the chapters....and did the little chapter problems that came with Kaplan. But it seems that the majority of the successful folks are doing more problems as they're reading along? IDK I'm kinda confused.
I think you have to find what works for you, and what's realistic given the amount of time that you have to study.
 
So its all depends on luck!!!! Ugh!!! What about B/B? My B/B score varies across all my exams lol. And CARS always depends on how motivated i am as you mentioned. Does it help to do the AAMC material twice? Plus you mentioned that no test company gets close to the real exam and you did much better on the real exam than on EK
I think that it depends on the time gap between material. I would do the Section Banks twice for sure. B/B has a rather harsh scale no matter what from what I saw on both my exams, especially the last one. I did find that EK's B/B was pretty comparable though and was the closest one by far for B/B. Once you do the AAMC material you will see what I'm talking about.
 
I watched many review videos and read books. After reading a chapter and taking notes I would do some associated practice problems (provided by Kaplan). I only took full practice exams once a week.

But thorough review of practice exams was great content review as well!!

Did you work through one subject at a time or did you do different subject and then take the practice full length weekly? thanks
 
Do you mind sharing your EK exam scores? And also for the second attempt did you retake any aamc material/was it indicative the second time around
Sorry about the late reply -- still figuring out how this website works! I only did Exam 1 and 2 from EK (got tired of spending money) and I got a 65% and 63% on those, respectively. I bought the "new" AAMC practice test and took it once with a 507, then studied from it by redoing questions I had gotten wrong, etc. I also used the older version of the AAMC practice test, but my access expired and I don't remember what my score was.
 
You mentioned teaching the subject to someone else really helps. I have a potential tutoring opportunity available.. would tutoring help in the way you mentioned or do you need some higher level analysis of the subject? I don't know I'm probably just overthinking this..my guess is yeah although it would be time consuming so I want to be 100% sure.

I mean, I studied with another person and we took turns teaching the material to each other when one of us didn't get it. If you're tutoring in the same subjects that are on the MCAT (biochem, orgo, genetics, etc) that that will probably be helpful, but if you're teaching like, High School Biology, I don't think that will help much (except for making you REALLY good at the basics I guess) Obviously teaching isn't to the exclusion of all other studying, but I did find that teaching the MCAT material to someone else helped me understand it (and remember it) about a million times better.
 
1) Your individual scores and composite score:
Total: 515 (94th %ile)
CP:
126 CARS: 131 BB: 128 P/S: 130

2) The study method used for each section:
All sections:
My main study method for all the sections was an in-house prep class by my university pre-health office based on EK materials. Students from the medical school here taught each of the subjects, and each had a different teaching style (e.g. the anatomy/phys guy did almost entirely lecture, while the biochem instructor had us doing PBL questions).

I started out by taking the AAMC scored test (now I think that was a mistake) to figure out where I stood. In retrospect, I would use any free FL exam for this and not the AAMC FL. I did my best, but I didn't worry about wrong answers too much, since this was the beginning. The day after, I went through every missed question and wrote down the content area from the MCAT outline so I would know what I needed to work on, and I made sure to hit those areas hard.

I read each segment/reading assignment at least once before class, preferably twice, doing the in-text questions as I went through and thoroughly checking answers & answer explanations. In the beginning I attempted to to the 30-min. quizzes when I got home after the class, but after 3 hours sitting in a windowless room it was unappealing at best, and I quit doing that after I fell asleep in my books a few times.

Near the end of Febuary (~12 weeks until MCAT) I started doing FL exams almost every other Saturday. For this I used free FLs for a while, then I ended up using the 3 FL exams I got from TPR when I bought one of their books. (Side note: try not to be discouraged by FL scores from non-AAMC sources.) I started working on the AAMC question packs a few weeks later, and tried to do a minimum of ~10 questions--more, if I could stand it--from each pack every week until I ran out.

It became such a habit that I almost forgot, but I also signed up for this MCAT Question of the Day thing from Kaplan, and when I was on my way to class I would open my email and do that every morning.

When I had about a month left before my test I took the AAMC Sample test and then began the CP section bank from AAMC. I was preparing for final exams and was battling some pretty severe symptoms of hypothyroidism at this point, so my motivation and energy were in the crapper and I did not get through all the section banks. In fact, the only section bank I did finish was CP. I would definitely not recommend that. You should really, really do all the section banks.

Two weeks out from the exam I took the AAMC FL #1 (again). I would recommend saving the AAMC FL exams until the end. They were a nice confidence boost from the TPR exams I had been taking.

CP: For chemistry I relied on the strategies outlined above, but I missed one of the physics classes, and physics is one of my weakest points. I couldn't digest the book chapters on my own, so I leaned heavily on Khan Academy videos, skimming the EK book and still doing the in-text questions.

CARS: Disclaimer: I didn't study much outside of the class I took for this section. In the CARS section of the class we basically did passages from Barron's and the EK 101 book from the old MCAT. We did the timed passages individually and then went through the questions as a group, with people explaining why they chose the answer they did, and then the instructor would lead us through the thought process to the right answer. A lot of times the answer explanation in the book is clear enough to do this on your own. We did these passages on paper, and I made a habit of marking key words: "best supports," "does NOT support," "weakens the argument," etc. in the question, which really helped me get rid of some questions I was missing for dumb reasons. There is a highlighter function on most online FL exams and the real MCAT, so this is a viable test strategy.

BB: In addition to EK materials, I used Khan Academy to review the details of some basic bio stuff, since Biology I was a few years ago (mitosis/meiosis, etc). I occasionally used the Khan Academy questions, but not much. I spent comparatively little time on MCAT-specific biochemistry, as I had just finished Biochem I and was currently taking Biochem II at my university during the semester I was studying for the MCAT.

P/S: Mostly EK, as noted above, but I also added in some supplemental reading from TPR. I went through the AAMC content outline and made sure I was able to explain a little bit about each item. Having to learn some detail was inevitable... Determining which details are important was easier because I had taken a psych class--e.g. Piaget, Pavlov, Freud, Milgram, Skinner, Bandura, Erikson, etc. pretty much always pop up in an intro psych course, and I knew I should be familiar with their experiments/findings. If I had not spent a lot of time in the social sciences for my major, I would have used TPR (or a similar, very detailed book) to familiarize myself with as much sociology terminology as possible, and tried to understand, not memorize, the meanings of those terms. The experimental portions of the test would be very difficult to interpret if I didn't have that background info.

3) What materials you used for each section(Kaplan, TPR, Examkrackers, AAMC, TBR, etc)
All sections:
Examkrackers 9th ed. full set
Kaplan MCAT Question of the Day
AAMC questions banks, section banks, sample test & FL 1.
Khan Academy (supplement to content review, not so much for practice questions)

Psych/Soc:
TPR Psychology/Sociology book + included FL exams (I bought this book from Barnes & Noble's website with a promo code & used it only minimally; I found that the EK material was a bit thin for this section, but TPR seemed way too detailed. I mostly bought the TPR book for the FL exams anyway.)

4) Which practice tests did you use? (Optional: include scores)
A bunch of free exams (didn't keep good records on these, sorry)
TPR FL exams (Sample, #1 and #2) -- I no longer have access to the score from the sample test, but Test 1: 504 (125, 127, 125, 127) and Test 2: 501 (123, 127, 125, 126)
AAMC Sample test (64%, 91%, 75%, 81%; roughly ~512 overall)
AAMC Practice Test 1 (511; 126, 130, 127, 128)

5) What was your undergraduate major?
Women's Studies. My social science background was very useful for CARS & P/S.

6) Any other tips you may have for those of us who still have this test lurking over us?
Take full advantage of AAMC materials. Do it all, maybe more than once. I really wish I had spent more time on the section banks. If I had, maybe I could have brought up CP and B/B and had a more balanced score.

If you've run out of practice questions and money to buy MCAT prep stuff, check with your pre-med or pre-health office--they may have old test prep books you can check out/borrow. Our office had the entire EK 1001 Questions/101 Passages collection available to be checked out by students, plus some Barron's stuff that I didn't get around to using.

Don't just review things that make you feel nice and confident. You should be saying "I should know this"/"Why can't I get this?"/"I F***ING HATE THIS S***!" while you study. It is a necessary evil.

On test day, don't worry too much about what you're eating, but go for somewhat familiar foods. I was WAY too hung up on this. I sprang for a hotel the night before to avoid driving 1.5 hrs. to the test center, so I used their free breakfast and had an bacon, egg & gouda breakfast sandwich (probs one of the best things I've ever eaten, btw), half a banana, and water. For lunch I had carrots with some garlic & herb cheese dip and some dark chocolate espresso beans. I had also brought a banana and a small bag of trail mix (dried cranberries, cashews, almonds, pistachios) which I did not touch. As far as drinks, I'm not a coffee drinker and I had been off other caffeinated drinks for almost a month, so I brought both water and Gatorade.

As a victim of really terrible timing, if you suspect something is wrong health-wise, are feeling sick or tired all the time, and/or are falling asleep in uncharacteristic places (i.e. in your prep books in the middle of the afternoon), go see a doctor to rule out a medical cause if you can afford it. My test anxiety balanced my insane fatigue right up to and including test day, making me weirdly calm, but my focus was at an all-time low. The importance of this test could not touch the black hole of exhaustion that was sucking away all my energy and motivation, and I barely studied in the last two weeks before the exam, which I deeply regret. Now that I'm being treated I feel 1000% better and I have no idea how I pulled off the MCAT in the state I was in.

7) How long did you study for the MCAT?
I started studying in January for my May 20 exam, so about ~4 months total, but I went easy for the first month and the last few weeks were a train wreck.

EDIT: Fixed paragraph breaks
 
Only because this advice may help some folks, and I can't be bothered to do real work for the moment:

1) Your individual scores and composite score
Total: 522, PS:128, CARS:130, BIO:132, PSY:132

2) The study method used for each section

I discuss this in 3 and 6.
3) What materials you used for each section(Kaplan, TPR, Examkrackers, AAMC, TBR, etc)
The reason I am posting is to advocate for an alternative, but succinct study method for those who were invested and did well in their coursework (i.e. have a strong foundation in biochemistry, chemistry, and maths). Other than practice questions and exams on the e-MCAT website I relied on one book religiously, Lehinger's Principles of Biochemistry. I think this would've been less useful for the previous MCAT, but I highly advise it for the new MCAT 2015. Outline the book, and review your notes, and outline again. The textbook does a brilliant job of building physics into chemistry into macroscopic biology via biochemical principles. There are even interesting medical sidenotes which frequently seem to make appearances in practice exams (i.e. physiology of haemoglobin form a thermodynamics perspective). And this is the nature of questions on both the PS and BIO sections. As you practice you'll see that, with some nuance in the nature of the questions emphasis on quantitative vs scientific though, topically these two sections are nearly identical now - albeit more orgo and spectroscopic topics make an appearance on the PS (still they will be cloaked in some biology no doubt). This really covers your groundwork for both the PS and the BIO basic knowledge, with the exception of circuits, simple machines, and kinematics. I supplemented the outlining of the book with responsive studying via the internet to incorrect question bank or practice test questions. And when I say responsive studying, I did not simply look up the answer to the missed question - I worded for an hour or so to re-cement the theory behind and surrounding the question. I would even do practice questions that one might find in a physics or chemistry book. I was extremely lucky on the psychology section having never taken sociology or psychology, I simply took some reddit notes of the material and outlined them, memorising the 60 or so scientists and their theories about how we function as individuals and a group - so I won't give advice on that one, and if I were to do so, in good conscious I could not suggest you take it.
4) Which practice tests did you use? (Optional: include scores)
AAMC FL - un-scored
AAMC FL - scored (Total: 514, PS:128, CARS:128, BIO:131, PSY:127 )
5) What was your undergraduate major?
Biochemistry & Maths
6) Any other tips you may have for those of us who still have this test lurking over us?
The use of Lehinger and responsive reading to incorrect questions advice should be caveated. For several years before the MCAT you should take a genuine interest in reading. I mean all kinds of reading. For instance, working through a scientific paper of some physical or biological topic that fascinates you a week would be extremely advisable. I recommend this, because four years of research resulted in my having to digest far more articles than that. And I could feel it helping me from the first cold practice test for the PS and BIO sections. Additionally, read the Washington Post, New York Times, the Economist, the Atlantic (I find their articles are particularly nuanced, insightful and well researched - good meaty practice for CARS), and any other news that isn't drivel. Finally, read books - non-fiction and fiction. Below I wrote three and a half weeks of studying. But that's not really accurate, that's only studying for the MCAT explicitly. There are countless hours of doing the hard, but enjoyable, work of reading up and following these topics over the course of the last eight years. These cannot be discounted, and I think they make difference when taking the exam. So for those that have the time left, I recommend you put in the effort now, so that painless love of learning translates into a much less painful intense study period for the exam. Discuss those articles and books you read with peers and understand what the author is saying before you critically appraise it. Read sympathetically, then critically. Trust me, it will be fun and will pay off come study time down the line.

7) How long did you study for the MCAT?
3.5 weeks (1.5 weeks full time for Psych and some biology)
 
  • Like
Reactions: 3 users
1) Composite score: 509 (82nd percentile) PS: 127 CARS: 127 BS: 128 Psych: 127

2) Practice Test: I took Kaplan FL exams and the official AAMC scored. I took the exam twice, once in January and once in May. I was hoping to hit it on the first shot but I fell into pitfalls the 1st time around that I will explicate upon below.

Kaplan FL Scores (Not in order)
Diagnostic: 488 *Not a typo :)* (Last August)
[Last October - January] 495, 496, 497, 499, 500
[Week before January exam] Official AAMC FL Practice: 508
[January Test] - Composite: 502 (60th percentile) PS: 125 CARS: 123 BS: 128 Psych: 126
[March-May] not in order 504, 498, 502, 506, 504
[May Test] - 509

3) Undergrad major: Biochemistry

4) Study Method/Schedule: THIS IS HOW I WENT FROM A 488 TO A 509; Before I explain, I just want to make it known that God was at the forefront of my preparation. To whomever is reading this, if I can ascend 21 points through Him, there’s no telling what you can do with Him and through Him. I would pray before every practice exam, I would pray everyday for God’s guidance in study, and I would put Him first and felt His supernatural guidance as a result. Try Him; He cannot fail you.

I began studying last September with the goal of reaching my target score in January. I enrolled in a Kaplan class that fall which was the fall of my junior year. I enrolled in the “in person” classroom plan which came with online classes through which numerous teachers would video chat with students everyday to go over every single concept on the test and all the concept videos were archived incase you would miss one [THE ONLINE VIDEOS WERE ONE OF THE HIGHLIGHTS OF MY STUDYING EXPERIENCE!! EVERY SINGLE PROFESSOR WAS LIKE THE COOLEST PERSON AND THEY IMPARTED CONFIDENCE AND BROKE DOWN EVERY SINGLE CONCEPT TO THE BASIC LEVEL THAT THE MCAT WILL BE TESTING MAKING IT MEMORABLE, EASY TO UNDERSTAND AND EFFECTIVE. THEY HELP ME REALIZED THAT AT THE END OF THE DAY THE AAMC IS TESTING FRESHMEN LEVEL SCIENCE WITH A LITTLE O.CHEM AND BIOCHEM. GO IN WITH THAT IN YOUR MIND KNOWING THAT WHAT REALLY GETS PEOPLE IS THE TRICKY MCAT LANGUAGE WHICH JUST TAKES PRACTICE TO GROW ADEPT AT DECIPHERING].

The classroom was more for strategy. In hindsight, I didn’t use any of their strategies because they simply did not work for me so I could have done without the in-class. However, the Professor I met through the class was golden and would just impart confidence which is a big part of success on this exam. If you are in a class, you have to take advantage of the Professor with questions because realize a prerequisite for getting an MCAT teaching job is a high mcat score. He actually scored in the 99th percentile and I asked him “how did you do that, and don’t bias your answer [haha]” and he told me he simply took many practice test and corrected them assiduously. So that’s what I began to do [Of course you should try and get all of the concepts under your belt before you start taking practice test left and right].

It came test time and I was feeling pumped! My official FL practice a week before was a 508 and I was feeling good. So it was test day, I got to the facility and began the Chem/Phys section. PITFALL NUMBER ONE: I let time escape me; I was dwelling on a lot of questions as opposed to strategically marking the unsure ones and moving on. At 1/3 of the exam, I had half of the exam remaining. PITFALL NUMBER TWO: The exam suddenly froze on me. I won’t lie, I was a little delighted because I thought I would get to restart the section. It took the 1.5 hours to resolve it, and it came back on right where I left off >__< this gave me an adrenaline rush though and I was able to finish the section. A month later, the score came in and I was not pleased. The biggest thing I learned was to not get sucked in by any question. Each question is worth the same so don’t waste minutes on a difficult question that you have a higher chance of getting wrong when there are questions following it that you could probably resolve in seconds. That "mark" button is your friend, just mark and come back. I learned to establish markers i.e. “I should have at least this many questions done before this time point.”

After I got my score back, I simply reviewed all the notes I compiled up till that point and took more practice test and graded them [my test grading resulted in me adding to the compiled notes]. I would thoroughly, thoroughly grade them all, looking for mental blunders, content confusion, all of that. I would make sure that every single question that I got wrong I would not miss again. I took one every Sunday closer to the 2nd exam and would grade them during the week. The last practice test I took was a week before my actual test.

How I boosted CARS score by 4 points: I found a strategy that worked for me and practiced it over and over again with at least one passage a day [except Saturdays, I would rest Saturdays as the 4th commandment instructs and as the Lord did when creating the earth] (mostly from the official aamc packet, some khan academy). The first time I took the exam, I didn’t have a single honed strategy, rather, I had a gross conglomeration of a bunch of different methods (Kaplan's, EK's, the guy up the block's) which did not serve me well. Find what works for you and hone it. This is what worked for me:

For Psych/Soc, I took a Psych class and reviewed my notes and my Kaplan MCAT book and also Khan academy! The videos were sooo helpful, there is even a khan academy app and I would listen to while I was doing whatever. [I use the Khan academy app prep for all sections] Studied the Kaplan book for Soc and googled whatever I was unfamiliar with that wasn't covered in the book that was listed under sociology in the outline that aamc provides for what is going to be on the test.

For Bio/Biochem, I was in a Biochem course and reviewed my notes but really relied heavily on the my online Kaplan class. Learned all the major pathways and what not, all amino acids, etc. Kaplan books were amazing for bio; the acronyms were soo amazing and quite comical.

For Chem/Phys relied heavily on flashcards for all of the equations. Helped soooo much! Use flashcards!!! Integrate not only the equations but an explanation of some sort. I used something called the leitner system (google it) but instead of three boxes I had one box with dividers in them for the different days.

6) Other tips: Be confident, be sure to get plenty of exercise while studying (improves mental function), eat a lot of berries and almonds (brain food), pack lots of food come test day, sleep well, and pray and have faith!!

7) How long did you study for the MCAT?
Intermittent:
September-early December (part time because classes)
Mid December to Late January (Full time)
March – May (part time - review of previously jotted notes, and practice test with extensive review weekly towards the tail end)

God Bless and pm with any questions. You got this, conquer it. Work harder today for a more relaxing and fulfilling tomorrow.
 
  • Like
Reactions: 6 users
1) Overall Score: 520 (98th percentile) PS: 131 CARS: 130 BS: 131 Psych: 128

2) Study Method: Physical Sciences has always been my weakest subject area, so I used both Kaplan and TPR books for these to make sure I fully covered the material since I had a poor foundation. I read each of the books a few times making notes as I went along and mostly focused on concepts, relationships between variables, and memorizing formulas for this section.
CARS I didn't study for other than the practice tests since I thought my time was better spent learning science content.
Biological sciences I spent most of my study time on; I read the Kaplan bio book multiple times until I felt comfortable explaining each topic, and also read through my notes from the biochemistry course I took the semester leading up to the test- I don't think I would have succeeded on this section without having taken that course, but I also did not have a science background so this may not apply to everyone. I memorized all the amino acids, hormones, enzymes, etc using flashcards and writing out different pathways multiple times.
Behavioral sciences I used TPR + Kaplan + glossary for intro to sociology text + Khan Academy videos to cover all my bases with it being a new section. I wrote out flashcards to memorize all of the terms in Kaplan. If I were taking the test again, I would probably have taken an intro psych/soc course or at least read a textbook as there were topics on my test that I had not seen in any of my prep materials.

3) Study Materials: Kaplan 7 book 2015 set**, Kaplan 528 book (didn't find this very helpful, it seemed mostly strategy based), TPR Chem/Phys, TPR Behavioral Sciences, Lehninger Biochemistry, intro to sociology textbook glossary, Kaplan online practice tests, AAMC FL, AAMC questions packs, AAMC Official Guide Questions, Khan Academy videos (mostly psychology and some biology)
** I didn't use the CARS book at all, and the orgo book I didn't spend much time on either. The other subjects I read through multiple times each until I was confident about the material and did all the practice questions. For physics, I picked out the formulas I thought would be most biologically relevant and memorized a sheet of them for the test; I found this more helpful than doing the practice problems in the Kaplan physics text.

4) Practice Tests: I used the online Kaplan FLs that came with the course, along with their initial half-length diagnostic. Also took a TPR FL, AAMC FL, and the AAMC Official Guide as a half length.

Kaplan FL Scores: (not in order)
Diagnostic: 504
FL 3: PS:125 CARS:128 BS:125 Psych:127 Total: 505
FL 4: PS:125 CARS:127 BS:125 Psych:127 Total: 504
FL 5: PS:124 CARS:126 BS:125 Psych:127 Total: 502
FL 8: PS:125 CARS:128 BS:126 Psych:124 Total: 503
FL 9: PS:125 CARS:128 BS:126 Psych:124 Total: 503

AAMC FL: PS: 78% CARS: 91% BS: 88% Psych: 83%

AAMC Official Guide: PS: 80% CARS: 90% BS: 76% Psych: 90%

5) Undergraduate major: Marketing, but I completed a post-bac program this year which included all pre-reqs

6) Tips: Can't speak for every test day, but my exam had a heavy biochem emphasis on both science sections, so I would recommend spending a larger amount of time studying those concepts than say orgo reactions (definitely memorize amino acids!). Physics was supposed to be de-emphasized vs. the old test, but I still had a number of physics questions, so I wouldn't blow off studying that either. Overall I think the Kaplan books did a great job of covering pretty much everything with the exception of behavioral sciences, so I think they are a pretty safe bet.

7) Time Spent Studying: I completed a 1-year post bac premed program, so I had taken all of the prerequisite coursework + biochem in the year leading up to the test (May 2015). I also was enrolled in the Kaplan on site course this spring, but only really used their book set for content & FLs as I didn't find their strategies helpful. I started studying MCAT specific materials around January, took my first diagnostic in February, and took a practice test roughly once per week starting around March. In the last two weeks before the test I stopped taking practice tests and spent about 8-10 hrs a day studying content, and took the AAMC FL roughly 1 week prior to the test date.


Good Job!!! Good luck on your studies!!!
 
1) Your individual scores and composite score
Composite score: 514 [129/128/128/129]
2) The study method used for each section
Honestly my biggest weakness was in psych/sociology. Being a bioengineering major really made my physical sciences/bio/biochem really fairly easy to catch up on or review. For CARS I have always been a huge history/politics geek (to keep myself sane outside of engineering), and CARS was always above 126 even on the kaplan tests I took. I really didn't study for this section at all. Taking philosophy/history/english classes with intense reading really is necessary to build the reading comprehension necessary to do well on this section. I really had to bang out psychology/sociology as a I didn't take any classes before taking the MCAT. I think this is the only section where you can get away with this and still do well. I spent about 2 weeks just reading the kaplan books and making flashcards for EVERY term in the book. I supplemented this with khan academy videos. It appeared to pay off. For chemistry/physics, I really felt like my undergraduate classes prepared me well, but learning how to problem solve and doing 3 years of research in an intense biochem lab really paid off. The main theme I see about the mcat is that the 3 months of studying isn't there to learn the material, it's there to solidify all the relevant knowledge from your undergraduate classes.
3) What materials you used for each section(Kaplan, TPR, Examkrackers, AAMC, TBR, etc)
Khan academy and Kaplan 7 book series. Make sure you do the practice problems offered by khan academy, they are VERY similar to the actual test questions. Watch videos in areas of weakness.
4) Which practice tests did you use? (Optional: include scores)
Kaplan 1: 498
Kaplan 2: 501
Kaplan 3: 503
AAMC unscored: 80%
AAMC scored: 510
5) What was your undergraduate major?
Bioengineering
6) Any other tips you may have for those of us who still have this test lurking over us?
I think the key for me was actually enjoying the material that was being studied. If you get up and aren't dreading the act of studying you will learn a lot more. The science behind medicine has always fascinated to me. A bad trap to get into is to ask yourself "why do I need to learn this?". A better Thing to tell yourself is "This maybe useful one day in my future medical career, and may help in the differential diagnosis of my future patients". That is how I got up every morning, ready to learn, and ready to give my studying time my best. Come test day, know that your feeling on the sections isn't necessarily correlated to your actual score. I left the test feeling defeated, and almost voided my score. When I saw my score I thought I had dyslexia, I couldn't believe it.
7) How long did you study for the MCAT?
2.5 months, but I took some vacations and acquainted myself with nature for a few days here and there. I also worked part-time as a lifeguard (I was able to study on breaks). PM me if you have questions!
 
  • Like
Reactions: 1 user
1) Your individual scores and composite score
130|128|128|128 (514 Composite)

2) The study method used for each section

First 4 weeks were spent on reading/content review and doing Anki flashcards for Biochem, Biology, and Psych/Sociology because those three/4 subjects comprise 73% of the PS/BS/BehS sections. Definitely memorize Amino Acids and chromatography/blotting types, trust me.

Week 5 was spent reviewing physics, general chem, and organic chem (more recent and less complex for me personally - see PS score).

Weeks 6-11 were spent doing practice problems/tests/section banks and reviewing those and all problems/topics that I missed and adding them to my Anki decks.

For PS I studied biochem mostly. I had taken Biochem in the spring, so I read the Khan academy book and my textbook on the topics we did not read in class. I also didn't really do much studying for general or organic chem because I was strong in those classes. I would just review topics as I missed them on practice sets and tests.

For CARS I started doing passages about 8 weeks or so out. Would do a couple passages a day, spent lots of time on them at first, then picked up my speed after the first couple weeks. Adopted a T-Chart strategy with a short sentence for every paragraph and/or new topic brought up in the passage and marked the paragraph it appeared in, writing it in either the left column (topic) or right column (opinion). When I adopted this strategy, I went from getting 50-60% correct to 75-85% correct.

For BS I read all of the chapters in the Kaplan books for Biochem and Biology and then reviewed missed topics after doing practice sets.

After taking practice questions/tests, I kept a log of every single one and the percent of questions I got correct. I usually did them in 20 question sets, and the practice tests I usually took all at one time.


3) What materials you used for each section(Kaplan, TPR, Examkrackers, AAMC, TBR, etc)

I used the Kaplan full 7 book series. I also bought the entire AAMC MCAT Bundle


4) Which practice tests did you use?

In the order I took them
AAMC Official Guide Diagnostic: (58%)
NEXT STEP Free full Length: (70%)
Kaplan 1: 499
Kaplan 2: 508
Kaplan 3: 505
AAMC Sample Test: (80%)
AAMC Section Bank
Physical Sciences: 61.5%
Biological Science: 59%
Behavioral Sciences: 74%
AAMC Full Length: 510 (79%)


5) What was your undergraduate major?

Bioengineering


6) Any other tips you may have for those of us who still have this test lurking over us?

Make a plan for every day of the 3 months (or however long you are studying for) of what you are going to study/what questions/test you are going to take each day.
Take all full length practice tests in a testing scenario and start at 8:00 a.m.
Definitely do the Section Banks and do them over again at least once!
Set a goal of getting 80+% right on all sections of any practice tests/sets that you do.

7) How long did you study for the MCAT?
11-12 weeks with 1-2 days off per week

Here is a document describing my exact studying process, that I found very helpful.
 

Attachments

  • MCAT Studying.docx
    121 KB · Views: 174
  • Like
Reactions: 1 user
I am SO happy that I can finally post in this thread. Posting here was a major motivation for me, I'm really not sure why, but it certainly worked.

I want to start this off by saying that I am not that smart, I didn't go to a fancy highly ranked undergrad, I didn't major in a natural science, and I certainly did not take an expensive prep course. I DID, however, work my A** off to do well on this test. This test will push you further than you thought you could go but if you take it in stride and do your best to stay calm, you will do fine!! I promise.

1) Score: 511 (86th Percentile); 128 Chem/Phys, 127 CARS, 128 Bio, 128 Psych/Soc
2) Study Method:

I started off much like everyone tells you to - Content Review. I initially intended on working through the Kaplan 7-book series one chapter a day, however, I found this to be a monumental waste of time. I found that I was reading through the chapters and being really bored and convincing myself that I knew that material cold. There wasn't nearly enough interaction for me. I am useless when I am just sitting there and reading. After the first week, I ramped up the content review to 4-5 chapters a day in a pattern that looked like this:

Monday - Physics Chapters n to n+5
Tuesday - Gen Chem Chapters n to n+5
Wednesday - Biochem Chapters n to n+5
Thursday - Bio Chapters n to n+5
Friday - Psych/Soc Chapters n to n+5
Repeat till finished.

I had just finished Orgo and that was a subject I was really good at so I did not review it at all. I made index cards in Anki while reading through each chapter. This took my about 8 hours a day and I was done after 4 weeks. I would intermittently study the index cards throughout the nights/weekends that I wasn't going through the chapters. I continued this until test day. I would never return to the books after this as I found them to be quite useless. I HATED THE PREP BOOKS AND I WISH I NEVER BOUGHT THEM.

After Content Review, I jumped right into taking practice exams, and all of AAMC's materials. I would take a test and then spend the next two days reviewing it. I didn't sit and analyze every question but I definitely attacked the ones that I did not understand. At this point I turned heavily to khan academy, ak lectures, and youtube videos. I learned so much through these channels, way more than I did through the Kaplan books. I spent the rest of my study time doing practice tests/questions/passages and reviewing what I got wrong. THIS WAS BY FAR THE BEST WAY TO STUDY FOR ME!

3) Materials: Kaplan 7 book series, NS tests, AAMC scored, AAMC unscored, Kaplan tests, Qpacks, Section Banks, TPR Demo, ANKI

4) Practice tests:

6/7/2016 - NS Diag - 502 -123/126/126/127
6/10/2016 - TPR Demo - 503 - 122/126/126/129
6/15/2016 - Kap FL 1 - 497 - 126/123/123/125
6/20/2016 - Kap FL 2 - 505 - 126/126/126/127
6/27/2016 - Kap FL 3 - 504 - 124/126/127/127
7/05/2016 - NS FL 1 - 509 - 126/127/128/128
7/08/2016 - NS FL 2 - 505 - 126/126/127/126
7/15/2016 - NS FL3 - 505 - 125/127/126/127
7/21/2016 - Unscored - 510(80%) - 125(63%)/128(87%)/127(78%)/130(93%)
7/25/2016 - NS FL 4 - 507 - 126/127/126/128
7/28/2016 - Scored - 512 - 127/128/127/130
8/04/2016 - MCAT - 511 - 128/127/128/128

6/13/2016 - Physics Qpack - 70%
6/14/2016 - Chemistry Qpack - 66%
6/23/2016 - Bio 1 Qpack - 73%
6/30/2016 - Bio 2 Qpack - 77%
7/11/2016 - BB SB - 51%
7/13/2016 - CP SB - 57%
7/18/2016 - Psy SB - 56%
7/19/2016 - Phys Qpack - 77%
7/20/2016 - Chem Qpack - 86%
7/23/2016 - Bio 1 Qpack - 88%
7/24/2016 - Bio 2 Qpack - 90%
7/26/2016 - CARS 1 Qpack - 66%
7/27/2016 - BB SB - 68%
7/31/2016 - CARS 2 Qpack - 76%
8/1/2016 - CP SB - 67%

5) Undergrad major: Psychology

6) Tips:

Here are some things that I did that I highly recommend doing:

I STAYED CALM AND PRACTICED GOOD TEST-TAKING ABILITIES!
I have my struggles with testing anxiety but I made sure to eliminate and kind of stress by having good test-taking abilities. I managed my time by writing how much time I should give myself on each passage. For example, the Chem/Phys section starts with 1 hour and 35 minutes to go, so I wrote 127 at the top of my scratch paper, representing 8 minutes for that passage. If I answered all of the questions for that passage, I would cross off that 127 and write how ever much time is left minus 8 minutes, so for example, 119. If I didn't know the answer immediately, I marked it and moved on. I always had about 30 minutes to go back at the end to answer the questions I skipped, which would be around 20 questions per section. The only section I didn't do this was CARS. But I did mark 10 minutes for every passage so I would write 120. I also made sure to target the answers that were definitely not correct (the MCAT always has 2 answers that can be eliminated outright) and I utilized the cross-off function.

I GOT GOOD(ISH) AT CARS!
This test is CARS. All of it. It's all reading comprehension. Get good at CARS. Seriously. READ EVERYTHING FOREVER EVERY DAY AND LEARN HOW TO READ GOOD!

I USED ALL OF AAMC'S MATERIALS!
There's nothing better than the AAMC materials, get your hands on all of it and study it well.

I TOOK WEEKENDS OFF!!
I cannot stress how important I think this is. I studied over the summer so I was not about to lose my entire summer to the MCAT. I went to the beach, concerts, hung out with friends, nothing too crazy but I made sure I went out. This allowed me to take my mind off the MCAT for two days and get back into it Monday morning with a fresh mind.

I STUDIED IN THE LIBRARY!
I cannot study at home, as there are too many distractions. I left the house at 8am and got to the library by 9am. I stayed there and studied until 5-6pm. After that I went home and was DONE (for the most part, maybe some flashcards). I made sure to take time away from this test to avoid burnout.

I DELETED ALL SOCIAL MEDIA FROM MY PHONE!
Seriously, stay away from distractions while studying. Focused studying is important and you won't learn anything if you're sitting there wondering was Jennifer or Tim did last night.

I PRE-PACKAGED HEALTHY MEALS!
I tried to eat as healthy as I could and made meals for myself for the week. I would bring a healthy lunch and snack with me to the library.

I GOT KAPLAN FL'S!
They're super hard. Because of this, get them!

I KEPT A HOLY GRAIL OF EXCEL SPREADSHEETS!
I tracked all of my progress and kept myself on schedule. I kept track of EVERYTHING!

Here are somethings that I wish I did:

I WISH I DIDN'T BUY ANY PREP BOOKS!!
They had no idea what would be on the test and presented such low-yield information. This test is about problem-solving, not really about content knowledge.

I WISH I UTILIZED THE AAMC SUBJECT LIST!
There is a list of all of the subjects tested on the MCAT that is published by the AAMC. Find it now. Go through it and go to the internet and learn. Learn from the internet. Have you been on the internet? It's got EVERYTHING. Seriously there are some great resources out there, I recommend the youtube channel AK Lectures HIGHLY! and Khan Academy is alright IMO.

I WISH I DID LESS CONTENT REVIEW!
Jump into practice asap. The best learning will come from practice materials. That's where you really see where your deficits are. You can read all day and convince yourself that you know it. You don't really know if you know it unless you're asked about it in some weird obscure way.

7) How long: 13 weeks.

I know there's more that I did but I can't think of anymore right now. GOOD LUCK! YOU CAN DO IT! This test is hard but it is NOT impossible!
 
Last edited:
  • Like
Reactions: 6 users
Use the template below:

1) Your individual scores and composite score
523 (131,130,130,132)

2) The study method used for each section
For CP it was all AAMC section bank. CARS practice tests. Bio reviewing books and practice tests. Psych, didn't study much.

3) What materials you used for each section(Kaplan, TPR, Examkrackers, AAMC, TBR, etc)
All of the above. Preferred order from best to worst: AAMC -> NS -> EK -> TBR -> Kaplan -> PR

4) Which practice tests did you use? (Optional: include scores)
AAMC: 513
For more see link below.

5) What was your undergraduate major?
Psychology

6) Any other tips you may have for those of us who still have this test lurking over us?
See link below

7) How long did you study for the MCAT?[/QUOTE]
Two months

See this link for more detailed advice:
http://forums.studentdoctor.net/threads/my-study-plan-for-mcat-523.1222623/
 
  • Like
Reactions: 1 users
1) Your individual scores and composite score
CP: 131 CARS: 127 BS: 130 PS: 130; Overall: 518 (97th percentile)

2) The study method used for each section
In the beginning of the summer (around mid-May), I began a very thorough review of all of the material. Then, I zoned in on specific topics that I knew I was fuzzy/unfamiliar with, and I studied those further.
Afterwards, I started doing a ton of passages (mostly from the Berkeley Review and EK). I took TBR prep class, and we were assigned homework (passages given to us in class and within the review books), which I compiled into a schedule and followed throughout the summer. As for specifics...

3) What materials you used for each section
I used TBR, TPR, and EK books.
TPR: This was what I used for my general, comprehensive overview. It's extremely dense and has lots of extraneous information, but it covers pretty much everything you need to know.
TBR: Where the Berkeley Review seriously shines is the sheer amount of practice problems that they give you. For each topic within each subject, there are roughly 15 passages and 100 total practice questions. Many of them are tricky, many will make you facepalm because you thought you had the right answer, and all of them are helpful. The Berkeley Review helped me TONS for discrete questions and learning to navigate passages. The questions will make you think very critically, and they're significantly more difficult than the problems you'll see on the real thing on test day. It really prepares you for those WTF questions that you might not expect, and you'll have a reliable test bank of knowledge ready to be used. Admittedly, I hardly read the books for content (extremely dense and overly detailed), but I imagine that it could really help with reinforcing concepts. One of my Berkeley Review teachers openly admitted that she (almost) never read the books, her entire study schedule was based on practice, practice, practice.
EK: I read all of these books to reinforce what I already knew. Furthermore, the 30 minute practice passages are EXTREMELY, EXTREMELY helpful. The questions are unreasonably difficult (much more difficult than AAMC-level questions), but they prepare you for the few difficult questions you might encounter on test day. The star of these practice passages are the passages themselves. After taking the AAMC FL and the real thing, EK did (in my opinion) the best job of capturing the essence of what the new MCAT passages are all about. Many of their passages are experiment/biochem heavy, and reading through them and getting used to them really helped me tackle those types of passages on the real deal.

C/P: Typically one of my weaker sections because I suck at physics (sound, doppler, light, optics, I hate it all). But, very thankfully, there was a strong biochem and gen chem emphasis on my exam (two of my strongest subjects). My preparation for physics mostly consisted of reviewing using TPR and EK, and using the practice problems from TBR and thoroughly trying to understand the answers. Since biochem and gen chem are my stronger suits, I didn't review much, mainly used practice problems as my way of studying those subjects.
CARS: Also one of my weaker sections, as reflected by my score. I did really well on CARS for some of my practice FLs, and really poorly on others. I can't say much for CARS besides practicing helps a lot, and really grasping the main idea of each passage is the key to doing well.
BS/B: Pure memorization when it came to review for bio. No other way around it, really. As for ochem and biochem, my courses in each subject really solidified my understanding of both, so they weren't so much of a problem for me. Again, I'd like to emphasize that doing practice problems will ultimately determine how well you know the material, being able to regurgitate information just doesn't cut it. Furthermore, Khan Academy videos does an amazing job of organizing all of the biochem topics you need to know. If you're rusty on gluconeogenesis, glycogenolysis, fatty acid oxidation, etc etc, then use their videos. Know enzymes. You never know what kind of discrete questions you could get. *hint hint*
P/S: Honestly, I'm not sure what happened here. I put off studying for P/S very last minute, I spent 3-4 days before my exam memorizing all of the theories and terms I had studied/reviewed from TPR. Furthermore, I went to the Khan Academy video website and watched pretty much every single video on topics that I was uncomfortable or unfamiliar with. I also did all of the P/S Khan Academy passages. Somehow, I pulled that 130 out of my butt. Be prepared for more sociology than you would expect/be familiar with. Also, I treated some of the passages as almost a CARS/P/S combination, and that worked for me. It seemed as though the passage writers had opinions which could be used to answer the questions.. Not sure if anyone else felt the same way. Also, I have heard that P/S has a very generous curve, but I'm not sure how accurate that assertion is.

4) Which practice tests did you use?

TPR (3 free ones with review books): 502, 503, 503
EK (FL 1, 2, 3): 82%, 71%, 73%
TBR: 509, 511, 513, 515
AAMC Official Guide: 26, 26, 22, 22
AAMC FL: 86%, 83%, 85%, 80%

5) What was your undergraduate major?
Neuroscience

6) Any other tips you may have for those of us who still have this test lurking over us?
**Disclaimer: I am a terrible standardized test taker. In the past, I've lacked the discipline to study in the long term, and I'm really good at cramming for exams and waiting until the last minute. All I can say is that if you're anything like me, suck it up and put in the effort. It will go a long way.**

Cannot emphasize enough, PRACTICE > CONTENT REVIEW. No matter how much you feel like you've mastered the material, it doesn't matter if you can't apply it to AAMC-like question types. This is why practice is miles more important if you have the general concepts down.

Get into a habit of waking up as early as you would on test day and doing practice problems/ tests to mirror testing conditions. This will help you immensely and eliminate some of the nerves that might be associated with test day.

Eat healthy and don't drink too much caffeine (less than you would on a normal day) on test day. The adrenaline is plenty sufficient to keep you awake and wired. I drank too much coffee and was shaking for half of the test.

Did I mention you should do practice problems? Because you should.

7) 3 months, from mid-May to mid-August. 2 hours of classes per week for 5-6 days per week, and 6-7 hours of studying each day on top of that.

Hi! thanks so much for your detailed reply. Its currently 2016 and I am preparing to take the MCAT in roughly 5 months (it all depends I haven't signed up yet). I have been studying for roughly three weeks. I am using EK and TBR (not TPR). I would really like to get your score! I find EK great for grounding me in the context/ tips/ their CARS is amazing ect. I have been using TBR for passages and inlecture probelms and after doing their gen chem it was so amazing I went ahead and began doing their lectures, too. I have the TBR from 2013 do you think I am disadvantaged here? I do problems frequently inlecture (foundational) and then at the commencement of each subject I do full blown passages till I want to die. I plan to buy EK 1001's for all subjects do all the problems for TBR and do FL's from all major companies (and AAMCAS of course). Were you fursrated when TBR passages asked material that you didn't study because they were no longer listed in the AAMCAS outline for 2015? Any help would be great as I am expiriencing frusrating with TBR's ochem due to the fact that it seems to cover material that is no longer on the MCAT 2015 changes. Thanks so much!
 
1) Your individual scores and composite score
523 (130/131/130/132)
2) The study method used for each section
Started in late May, went to prep course classes about 3x a week for four hours a day (I skipped most of the CARS lessons because they weren't personally very helpful after the first class), started practice tests a little less than a month prior to the test date (Aug 5th).
3) What materials you used for each section(Kaplan, TPR, Examkrackers, AAMC, TBR, etc)
Prep101 MCAT course (AKA 101 Passages in Verbal Reasoning, 1001 Questions in MCAT Biology, EK books, NS FL practice exams + 1AAMC FL)
4) Which practice tests did you use? (Optional: include scores)
4 NS practice tests: 511 average (127/128/127/129, 127/128/128/128, 128/127/127/129, 128/128/127/129)
Scored AAMC practice test: 521 (130/131/130/130)
5) What was your undergraduate major?
Psychology with a minor in sociology (P/S section was a blessing tbh)
6) Any other tips you may have for those of us who still have this test lurking over us?
Don't get burnt out, try to space out your studying as much as possible and most of all, do as many practice tests as you can and /review all your mistakes in detail/. Plan something fun on the day before the test.
7) How long did you study for the MCAT?
Roughly 2.5 months.
 
  • Like
Reactions: 2 users
Top